Формула а минус б в кубе: § Формулы сокращённого умножения. Разность квадратов. Квадрат суммы. Куб суммы. Разность кубов и другие формулы

Содержание

§ Формулы сокращённого умножения. Разность квадратов. Квадрат суммы. Куб суммы. Разность кубов и другие формулы

При расчёте алгебраических многочленов для упрощения вычислений используются формулы сокращенного умножения. Всего таких формул семь. Их все необходимо знать наизусть.

Следует также помнить, что вместо «a» и «b» в формулах могут стоять как числа, так и любые другие алгебраические многочлены.

Разность квадратов

Запомните!

Разность квадратов двух чисел равна произведению разности этих чисел и их суммы.

a2 − b2 = (a − b)(a + b)

Примеры:

  • 152 − 22 = (15 − 2)(15 + 2) = 13 · 17 = 221
  • 9a2 − 4b2с2 = (3a − 2bc)(3a + 2bc)

Квадрат суммы

Запомните!

Квадрат суммы двух чисел равен квадрату первого числа плюс удвоенное произведение первого числа на второе плюс квадрат второго числа.

(a + b)2 = a2 + 2ab + b2

Обратите внимание, что с помощью этой формулы сокращённого умножения легко находить квадраты больших чисел, не используя калькулятор или умножение в столбик. Поясним на примере:

Найти 1122.

  • Разложим 112 на сумму чисел, чьи квадраты мы хорошо помним.
    112 = 100 + 1
  • Запишем сумму чисел в скобки и поставим над скобками квадрат.
    1122 = (100 + 12)2
  • Воспользуемся формулой квадрата суммы:
    1122 = (100 + 12)2 = 1002 + 2 · 100 · 12 + 122 = 10 000 + 2 400 + 144 = 12 544

Помните, что формула квадрат суммы также справедлива для любых алгебраических многочленов.

  • (8a + с)2 = 64a2 + 16ac + c2

Предостережение!

(a + b)2 не равно (a2 + b2)

Квадрат разности

Запомните!

Квадрат разности двух чисел равен квадрату первого числа минус удвоенное произведение первого на второе плюс квадрат второго числа.

(a − b)2 = a2 − 2ab + b2

Также стоит запомнить весьма полезное преобразование:

(a − b)2 = (b − a)2

Формула выше доказывается простым раскрытием скобок:

(a − b)2 = a2 −2ab + b2 = b2 − 2ab + a2 = (b − a)2

Куб суммы

Запомните!

Куб суммы двух чисел равен кубу первого числа плюс утроенное произведение квадрата первого числа на второе плюс утроенное произведение первого на квадрат второго плюс куб второго.

(a + b)3 = a3 + 3a2b + 3ab2 + b3

Как запомнить куб суммы

Запомнить эту «страшную» на вид формулу довольно просто.

  • Выучите, что в начале идёт «a3».
  • Два многочлена посередине имеют коэффициенты 3.
  • Вспомним, что любое число в нулевой степени есть 1. (a0 = 1, b0 = 1). Легко заметить, что в формуле идёт понижение степени «a» и увеличение степени «b». В этом можно убедиться:
    (a + b)3 = a3b0 + 3a2b1 + 3a1b2 + b3
    a0 = a3 + 3a2b + 3ab2 + b3

Предостережение!

(a + b)3 не равно a3 + b3

Куб разности

Запомните!

Куб разности двух чисел равен кубу первого числа минус утроенное произведение квадрата первого числа на второе плюс утроенное произведение первого числа на квадрат второго минус куб второго.

(a − b)3 = a3 − 3a2b + 3ab2 − b3

Запоминается эта формула как и предыдущая, но только с учётом чередования знаков «+» и «−». Перед первым членом «a3 » стоит «+» (по правилам математики мы его не пишем). Значит, перед следующим членом будет стоять «−», затем опять «+» и т.д.

(a − b)3 = + a3 − 3a2b + 3ab2 − b3 = a3 − 3a2b + 3ab2 − b3

Сумма кубов

Не путать с кубом суммы!

Запомните!

Сумма кубов равна произведению суммы двух чисел на неполный квадрат разности.

a3 + b3 = (a + b)(a2 − ab + b2)

Сумма кубов — это произведение двух скобок.

  • Первая скобка — сумма двух чисел.
  • Вторая скобка — неполный квадрат разности чисел. Неполным квадратом разности называют выражение:
    (a2− ab + b2)
    Данный квадрат неполный, так как посередине вместо удвоенного произведения обычное произведение чисел.

Разность кубов

Не путать с кубом разности!

Запомните!

Разность кубов равна произведению разности двух чисел на неполный квадрат суммы.

a3 − b3 = (a − b)(a2 + ab + b2)

Будьте внимательны при записи знаков.

Применение формул сокращенного умножения

Следует помнить, что все формулы, приведённые выше, используется также и справа налево.

Многие примеры в учебниках рассчитаны на то, что вы с помощью формул соберёте многочлен обратно.

Примеры:

  • a2 + 2a + 1 = (a + 1)2
  • (aс − 4b)(ac + 4b) = a2c2 − 16b2

Таблицу со всеми формулами сокращённого умножения вы можете скачать в разделе «Шпаргалки».


А плюс в в кубе формула. Возведение в куб. Формулы сокращенного умножения

Формулы сокращенного выражения очень часто применяются на практике, так что их все желательно выучить наизусть. До этого момента нам будет служить верой и правдой , которую мы рекомендуем распечатать и все время держать перед глазами:

Первые четыре формулы из составленной таблицы формул сокращенного умножения позволяют возводить в квадрат и куб сумму или разность двух выражений. Пятая предназначена для краткого умножения разности и суммы двух выражений. А шестая и седьмая формулы используются для умножения суммы двух выражений a и b на их неполный квадрат разности (так называют выражение вида a 2 −a·b+b 2 ) и разности двух выражений a и b на неполный квадрат их суммы (a 2 +a·b+b 2 ) соответственно.

Стоит отдельно заметить, что каждое равенство в таблице представляет собой тождество . Этим объясняется, почему формулы сокращенного умножения еще называют тождествами сокращенного умножения.

При решении примеров, особенно в которых имеет место разложение многочлена на множители , ФСУ часто используют в виде с переставленными местами левыми и правыми частями:


Три последних тождества в таблице имеют свои названия. Формула a 2 −b 2 =(a−b)·(a+b) называется формулой разности квадратов , a 3 +b 3 =(a+b)·(a 2 −a·b+b 2 ) — формулой суммы кубов , а a 3 −b 3 =(a−b)·(a 2 +a·b+b 2 ) — формулой разности кубов . Обратите внимание, что соответствующим формулам с переставленными частями из предыдущей таблицы фсу мы никак не назвали.

Дополнительные формулы

В таблицу формул сокращенного умножения не помешает добавить еще несколько тождеств.

Сферы применения формул сокращенного умножения (фсу) и примеры

Основное предназначение формул сокращенного умножения (фсу) объясняется их названием, то есть, оно состоит в кратком умножении выражений. Однако сфера применения ФСУ намного шире, и не ограничивается кратким умножением. Перечислим основные направления.

Несомненно, центральное приложение формулы сокращенного умножения нашли в выполнении тождественных преобразований выражений . Наиболее часто эти формулы используются в процессе

упрощения выражений .

Пример.

Упростите выражение 9·y−(1+3·y) 2 .

Решение.

В данном выражении возведение в квадрат можно выполнить сокращенно, имеем 9·y−(1+3·y) 2 =9·y−(1 2 +2·1·3·y+(3·y) 2) . Остается лишь раскрыть скобки и привести подобные члены: 9·y−(1 2 +2·1·3·y+(3·y) 2)= 9·y−1−6·y−9·y 2 =3·y−1−9·y 2 .

В предыдущем уроке мы разобрались с разложением на множители. Освоили два способа: вынесение общего множителя за скобки и группировку. В этом уроке — следующий мощный способ: формулы сокращённого умножения . В краткой записи — ФСУ.

Формулы сокращённого умножения (квадрат суммы и разности, куб суммы и разности, разность квадратов, сумма и разность кубов) крайне необходимы во всех разделах математики. Они применяются в упрощении выражений, решении уравнений, умножении многочленов, сокращении дробей, решении интегралов и т.

д. и т.п. Короче, есть все основания разобраться с ними. Понять откуда они берутся, зачем они нужны, как их запомнить и как применять.

Разбираемся?)

Откуда берутся формулы сокращённого умножения?

Равенства 6 и 7 записаны не очень привычно. Как бы наоборот. Это специально.) Любое равенство работает как слева направо, так и справа налево. В такой записи понятнее, откуда берутся ФСУ.

Они берутся из умножения.) Например:

(a+b) 2 =(a+b)(a+b)=a 2 +ab+ba+b 2 =a 2 +2ab+b 2

Вот и всё, никаких научных хитростей. Просто перемножаем скобки и приводим подобные. Так получаются все формулы сокращённого умножения. Сокращённое умножение — это потому, что в самих формулах нет перемножения скобок и приведения подобных. Сокращены.) Сразу дан результат.

ФСУ нужно знать наизусть. Без первых трёх можно не мечтать о тройке, без остальных — о четвёрке с пятёркой.)

Зачем нужны формулы сокращённого умножения?

Есть две причины, выучить, даже зазубрить эти формулы.

3 = 8 .

Примеры для решения:

Возведение в степень презентация

Презентация по возведению в степень, рассчитанную на семиклассников. Презентация может разъяснить некоторые непонятные моменты, но, вероятно, таких моментов не будет благодаря нашей статье.

Итог

Мы рассмотрели лишь верхушку айсберга, чтобы понять математику лучше — записывайтесь на наш курс: Ускоряем устный счет — НЕ ментальная арифметика.

Из курса вы не просто узнаете десятки приемов для упрощенного и быстрого умножения, сложения, умножения, деления, высчитывания процентов, но и отработаете их в специальных заданиях и развивающих играх! Устный счет тоже требует много внимания и концентрации, которые активно тренируются при решении интересных задач.

Формулы сокращенного умножения.

Изучение формул сокращенного умножения: квадрата суммы и квадрата разности двух выражений; разности квадратов двух выражений; куба суммы и куба разности двух выражений; суммы и разности кубов двух выражений.

Применение формул сокращенного умножения при решении примеров.

Для упрощения выражений, разложения многочленов на множители, приведения многочленов к стандартному виду используются формулы сокращенного умножения. Формулы сокращенного умножения нужно знать наизусть .

Пусть а, b R. Тогда:

1. Квадрат суммы двух выражений равен квадрату первого выражения плюс удвоенное произведение первого выражения на второе плюс квадрат второго выражения.

(a + b) 2 = a 2 + 2ab + b 2

2. Квадрат разности двух выражений равен квадрату первого выражения минус удвоенное произведение первого выражения на второе плюс квадрат второго выражения.

(a — b) 2 = a 2 — 2ab + b 2

3. Разность квадратов двух выражений равна произведению разности этих выражений и их суммы.

a 2 — b 2 = (a -b) (a+b)

4. Куб суммы двух выражений равен кубу первого выражения плюс утроенное произведение квадрата первого выражения на второе плюс утроенное произведение первого выражения на квадрат второго плюс куб второго выражения.

(a + b) 3 = a 3 + 3a 2 b + 3ab 2 + b 3

5. Куб разности двух выражений равен кубу первого выражения минус утроенное произведение квадрата первого выражения на второе плюс утроенное произведение первого выражения на квадрат второго минус куб второго выражения.

(a — b) 3 = a 3 — 3a 2 b + 3ab 2 — b 3

6. Сумма кубов двух выражений равна произведению суммы первого и второго выражения на неполный квадрат разности этих выражений.

a 3 + b 3 = (a + b) (a 2 — ab + b 2)

7. Разность кубов двух выражений равна произведению разности первого и второго выражения на неполный квадрат суммы этих выражений.

a 3 — b 3 = (a — b) (a 2 + ab + b 2)

Применение формул сокращенного умножения при решении примеров.

Пример 1.

Вычислить

а) Используя формулу квадрата суммы двух выражений, имеем

(40+1) 2 = 40 2 + 2 · 40 · 1 + 1 2 = 1600 + 80 + 1 = 1681

б) Используя формулу квадрата разности двух выражений, получим

98 2 = (100 – 2) 2 = 100 2 — 2 · 100 · 2 + 2 2 = 10000 – 400 + 4 = 9604

Пример 2.

Вычислить

Используя формулу разности квадратов двух выражений, получим

Пример 3.

Упростить выражение

(х — у) 2 + (х + у) 2

Воспользуемся формулами квадрата суммы и квадрата разности двух выражений

(х — у) 2 + (х + у) 2 = х 2 — 2ху + у 2 + х 2 + 2ху + у 2 = 2х 2 + 2у 2

Формулы сокращенного умножения в одной таблице:

(a + b) 2 = a 2 + 2ab + b 2
(a — b) 2 = a 2 — 2ab + b 2
a 2 — b 2 = (a — b) (a+b)
(a + b) 3 = a 3 + 3a 2 b + 3ab 2 + b 3
(a — b) 3 = a 3 — 3a 2 b + 3ab 2 — b 3
a 3 + b 3 = (a + b) (a 2 — ab + b 2)
a 3 — b 3 = (a — b) (a 2 + ab + b 2)

правила применения формул сокращенного умножения. Формулы сокращенного умножения Чему равна разность кубов двух выражений

Формулы или правила сокращенного умножения используются в арифметике, а точнее — в алгебре, для более быстрого процесса вычисления больших алгебраических выражений. Сами же формулы получены из существующих в алгебре правил для умножения нескольких многочленов.

Использование данных формул обеспечивает достаточно оперативное решение различных математических задач, а также помогает осуществлять упрощение выражений. Правила алгебраических преобразований позволяют выполнять некоторые манипуляции с выражениями, следуя которым можно получить в левой части равенства выражение, стоящее в правой части, или преобразовать правую часть равенства (чтобы получить выражение, стоящее в левой части после знака равенства).

Удобно знать формулы, применяемые для сокращенного умножения, на память, так как они нередко используются при решении задач и уравнений. Ниже перечислены основные формулы, входящие в данный список, и их наименование.

Квадрат суммы

Чтобы вычислить квадрат суммы, необходимо найти сумму, состоящую из квадрата первого слагаемого, удвоенного произведения первого слагаемого на второе и квадрата второго. В виде выражения данное правило записывается следующим образом: (а + с)² = a² + 2ас + с².

Квадрат разности

Чтобы вычислить квадрат разности, необходимо вычислить сумму, состоящую из квадрата первого числа, удвоенного произведения первого числа на второе (взятое с противоположным знаком) и квадрата второго числа. В виде выражения данное правило выглядит следующим образом: (а — с)² = а² — 2ас + с².

Разность квадратов

Формула разности двух чисел, возведенных в квадрат, равна произведению суммы этих чисел на их разность. В виде выражения данное правило выглядит следующим образом: a² — с² = (a + с)·(a — с).

Куб суммы

Чтобы вычислить куб суммы двух слагаемых, необходимо вычислить сумму, состоящую из куба первого слагаемого, утроенного произведения квадрата первого слагаемого и второго, утроенного произведения первого слагаемого и второго в квадрате, а также куба второго слагаемого. В виде выражения данное правило выглядит следующим образом: (а + с)³ = а³ + 3а²с + 3ас² + с³.

Сумма кубов

Согласно формуле, приравнивается к произведению суммы данных слагаемых на их неполный квадрат разности. В виде выражения данное правило выглядит следующим образом: а³ + с³ = (а + с)·(а² — ас + с²).

Пример. Необходимо вычислить объем фигуры, которая образована сложением двух кубов. Известны лишь величины их сторон.

Если значения сторон небольшие, то выполнить вычисления просто.

Если же длины сторон выражаются в громоздких числах, то в этом случае проще применить формулу «Сумма кубов», которая значительно упростит вычисления.

Куб разности

Выражение для кубической разности звучит так: как сумма третьей степени первого члена, утроенного отрицательного произведения квадрата первого члена на второй, утроенного произведения первого члена на квадрат второго и отрицательного куба второго члена. В виде математического выражения куб разности выглядит следующим образом: (а — с)³ = а³ — 3а²с + 3ас² — с³.

Разность кубов

Формула разности кубов отличается от суммы кубов лишь одним знаком. Таким образом, разность кубов — формула, равная произведению разности данных чисел на их неполный квадрат суммы. В виде разность кубов выглядит следующим образом: а 3 — с 3 = (а — с)(а 2 + ас + с 2).

Пример. Необходимо вычислить объем фигуры, которая останется после вычитания из объема синего куба объемной фигуры желтого цвета, которая также является кубом. Известна лишь величина стороны маленького и большого куба.

Если значения сторон небольшие, то вычисления довольно просты. А если длины сторон выражаются в значительных числах, то стоит применить формулу, озаглавленную «Разность кубов» (или «Куб разности»), которае значительно упростит вычисления.

Формулы сокращенного умножения (ФСУ) применяются для возведения в степень и умножения чисел и выражений. Часто эти формулы позволяют произвести вычисления более компактно и быстро.

В данной статье мы перечислим основные формулы сокращенного умножения, сгруппируем их в таблицу, рассмотрим примеры использования этих формул, а также остановимся на принципах доказательств формул сокращенного умножения.

Впервые тема ФСУ рассматривается в рамках курса «Алгебра» за 7 класс. Приведем ниже 7 основных формул.

Формулы сокращенного умножения

  1. формула квадрата суммы: a + b 2 = a 2 + 2 a b + b 2
  2. формула квадрата разности: a — b 2 = a 2 — 2 a b + b 2
  3. формула куба суммы: a + b 3 = a 3 + 3 a 2 b + 3 a b 2 + b 3
  4. формула куба разности: a — b 3 = a 3 — 3 a 2 b + 3 a b 2 — b 3
  5. формула разности квадратов: a 2 — b 2 = a — b a + b
  6. формула суммы кубов: a 3 + b 3 = a + b a 2 — a b + b 2
  7. формула разности кубов: a 3 — b 3 = a — b a 2 + a b + b 2

Буквами a, b, c в данных выражениях могут быть любые числа, переменные или выражения. Для удобства использования лучше выучить семь основных формул наизусть. Сведем их в таблицу и приведем ниже, обведя рамкой.

Первые четыре формулы позволяют вычислять соответственно квадрат или куб суммы или разности двух выражений.

Пятая формула вычисляет разность квадратов выражений путем произведения их суммы и разности.

Шестая и седьмая формулы — соответственно умножение суммы и разности выражений на неполный квадрат разности и неполный квадрат суммы.

Формула сокращенного умножения иногда еще называют тождествами сокращенного умножения. В этом нет ничего удивительного, так как каждое равенство представляет собой тождество.

При решении практических примеров часто используют формулы сокращенного умножения с переставленными местами левыми и правыми частями. Это особенно удобно, когда имеет место разложение многочлена на множители.

Дополнительные формулы сокращенного умножения

Не будем ограничиваться курсом 7 класса по алгебре и добавим в нашу таблицу ФСУ еще несколько формул.

Во-первых, рассмотрим формулу бинома Ньютона.

a + b n = C n 0 · a n + C n 1 · a n — 1 · b + C n 2 · a n — 2 · b 2 + . . + C n n — 1 · a · b n — 1 + C n n · b n

Здесь C n k — биномиальные коэффициенты, которые стоят в строке под номером n в треугольнике паскаля. Биномиальные коэффициенты вычисляются по формуле:

C n k = n ! k ! · (n — k) ! = n (n — 1) (n — 2) . . (n — (k — 1)) k !

Как видим, ФСУ для квадрата и куба разности и суммы — это частный случай формулы бинома Ньютона при n=2 и n=3соответственно.

Но что, если слагаемых в сумме, которую нужно возвести в степень, больше, чем два? Полезной будет формула квадрата суммы трех, четырех и более слагаемых.

a 1 + a 2 + . . + a n 2 = a 1 2 + a 2 2 + . . + a n 2 + 2 a 1 a 2 + 2 a 1 a 3 + . . + 2 a 1 a n + 2 a 2 a 3 + 2 a 2 a 4 + . . + 2 a 2 a n + 2 a n — 1 a n

Еще одна формула, которая может пригодится — формула формула разности n-ых степеней двух слагаемых.

a n — b n = a — b a n — 1 + a n — 2 b + a n — 3 b 2 + . . + a 2 b n — 2 + b n — 1

Эту формулу обычно разделяют на две формулы — соответственно для четных и нечетных степеней.

Для четных показателей 2m:

a 2 m — b 2 m = a 2 — b 2 a 2 m — 2 + a 2 m — 4 b 2 + a 2 m — 6 b 4 + . . + b 2 m — 2

Для нечетных показателей 2m+1:

a 2 m + 1 — b 2 m + 1 = a 2 — b 2 a 2 m + a 2 m — 1 b + a 2 m — 2 b 2 + . . + b 2 m

Формулы разности квадратов и разности кубов, как вы догадались, являются частными случаями этой формулы при n = 2 и n = 3 соответственно. Для разности кубов b также заменяется на — b .

Как читать формулы сокращенного умножения?

Дадим соответствующие формулировки для каждой формулы, но сначала разберемся с принципом чтения формул. Удобнее всего делать это на примере. Возьмем самую первую формулу квадрата суммы двух чисел.

a + b 2 = a 2 + 2 a b + b 2 .

Говорят: квадрат суммы двух выражений a и b равен сумме квадрата первого выражения, удвоенного произведения выражений и квадрата второго выражения.

Все остальные формулы читаются аналогично. Для квадрата разности a — b 2 = a 2 — 2 a b + b 2 запишем:

квадрат разности двух выражений a и b равен сумме квадратов этих выражений минус удвоенное произведение первого и второго выражения.

Прочитаем формулу a + b 3 = a 3 + 3 a 2 b + 3 a b 2 + b 3 . Куб суммы двух выражений a и b равен сумме кубов этих выражений, утроенного произведения квадрата первого выражения на второе и утроенного произведения квадрата второго выражения на первое выражение.

Переходим к чтению формулы для разности кубов a — b 3 = a 3 — 3 a 2 b + 3 a b 2 — b 3 . Куб разности двух выражений a и b равен кубу первого выражения минус утроенное произведение квадрата первого выражения на второе, плюс утроенное произведение квадрата второго выражения на первое выражение, минус куб второго выражения.

Пятая формула a 2 — b 2 = a — b a + b (разность квадратов) читается так: разность квадратов двух выражений равна произведению разности и суммы двух выражений.

Выражения типа a 2 + a b + b 2 и a 2 — a b + b 2 для удобства называют соответственно неполным квадратом суммы и неполным квадратом разности.

С учетом этого, формулы суммы и разности кубов прочитаются так:

Сумма кубов двух выражений равна произведению суммы этих выражений на неполный квадрат их разности.

Разность кубов двух выражений равна произведению разности этих выражений на неполный квадрат их суммы.

Доказательство ФСУ

Доказать ФСУ довольно просто. Основываясь на свойствах умножения, проведем умножение частей формул в скобках.

Для примера рассмотрим формулу квадрата разности.

a — b 2 = a 2 — 2 a b + b 2 .

Чтобы возвести выражение во вторую степень нужно это выражение умножить само на себя.

a — b 2 = a — b a — b .

Раскроем скобки:

a — b a — b = a 2 — a b — b a + b 2 = a 2 — 2 a b + b 2 .

Формула доказана. Остальные ФСУ доказываются аналогично.

Примеры применения ФСУ

Цель использования формул сокращенного умножения — быстрое и краткое умножение и возведение выражений в степень. Однако, это не вся сфера применения ФСУ. Они широко используются при сокращении выражений, сокращении дробей, разложении многочленов на множители. Приведем примеры.

Пример 1. ФСУ

Упростим выражение 9 y — (1 + 3 y) 2 .

Применим формулу суммы квадратов и получим:

9 y — (1 + 3 y) 2 = 9 y — (1 + 6 y + 9 y 2) = 9 y — 1 — 6 y — 9 y 2 = 3 y — 1 — 9 y 2

Пример 2. ФСУ

Сократим дробь 8 x 3 — z 6 4 x 2 — z 4 .

Замечаем, что выражение в числителе — разность кубов, а в знаменателе — разность квадратов.

8 x 3 — z 6 4 x 2 — z 4 = 2 x — z (4 x 2 + 2 x z + z 4) 2 x — z 2 x + z .

Сокращаем и получаем:

8 x 3 — z 6 4 x 2 — z 4 = (4 x 2 + 2 x z + z 4) 2 x + z

Также ФСУ помогают вычислять значения выражений. Главное — уметь заметить, где применить формулу. Покажем это на примере.

Возведем в квадрат число 79 . Вместо громоздких вычислений, запишем:

79 = 80 — 1 ; 79 2 = 80 — 1 2 = 6400 — 160 + 1 = 6241 .

Казалось бы, сложное вычисление проведено быстро всего лишь с использованием формул сокращенного умножения и таблицы умножения.

Еще один важный момент — выделение квадрата двучлена. Выражение 4 x 2 + 4 x — 3 можно преобразовать в вид 2 x 2 + 2 · 2 · x · 1 + 1 2 — 4 = 2 x + 1 2 — 4 . Такие преобразования широко используются в интегрировании.

Если вы заметили ошибку в тексте, пожалуйста, выделите её и нажмите Ctrl+Enter

В предыдущих уроках мы рассмотрели два способа разложения многочлена на множители: вынесение общего множителя за скобки и способ группировки .

В этом уроке мы рассмотрим еще один способ разложения многочлена на множители с применением формул сокращённого умножения .

Рекомендуем каждую формулу прописать не менее 12 раз. Для лучшего запоминания выпишите все формулы сокращённого умножения себе на небольшую шпаргалку .

Вспомним, как выглядит формула разности кубов.

a 3 − b 3 = (a − b)(a 2 + ab + b 2)

Формула разности кубов не очень проста для запоминания, поэтому рекомендуем использовать специальный способ для её запоминания.

Важно понимать, что любая формула сокращённого умножения действует и в обратную сторону .

(a − b)(a 2 + ab + b 2) = a 3 − b 3

Рассмотрим пример. Необходимо разложить на множители разность кубов.

Обратим внимание, что «27а 3 » — это «(3а) 3 », значит, для формулы разности кубов вместо «a » мы используем «3a ».

Используем формулу разности кубов. На месте «a 3 » у нас стоит «27a 3 », а на месте «b 3 », как и в формуле, стоит «b 3 ».

Применение разности кубов в обратную сторону

Рассмотрим другой пример. Требуется преобразовать произведение многочленов в разность кубов, используя формулу сокращенного умножения.

Обратите внимание, что произведение многочленов «(x − 1)(x 2 + x + 1) » напоминает правую часть формулы разности кубов «», только вместо «a » стоит «x », а на месте «b » стоит «1 ».

Используем для «(x − 1)(x 2 + x + 1) » формулу разности кубов в обратную сторону.


Рассмотрим пример сложнее. 2\right)\]

Выражение в кубе. Формулы сокращенного умножения

Математические выражения (формулы) сокращённого умножения (квадрат суммы и разности, куб суммы и разности, разность квадратов, сумма и разность кубов) крайне не заменимы во многих областях точных наук. Эти 7 символьных записей не заменимы при упрощении выражений, решении уравнений, при умножении многочленов, сокращении дробей , решении интегралов и многом другом. А значит будет очень полезно разобраться как они получаются, для чего они нужны, и самое главное, как их запомнить и потом применять. Потом применяя формулы сокращенного умножения на практике самым сложным будет увидеть, что есть х и что есть у. Очевидно, что никаких ограничений для a и b нет, а значит это могут быть любые числовые или буквенные выражения.

И так вот они:

Первая х 2 — у 2 = (х — у) (х+у) .Чтобы рассчитать разность квадратов двух выражений надо перемножить разности этих выражений на их суммы.

Вторая (х + у) 2 = х 2 + 2ху + у 2 . Чтобы найти квадрат суммы двух выражений нужно к квадрату первого выражения прибавить удвоенное произведение первого выражения на второе плюс квадрат второго выражения.

Третья (х — у) 2 = х 2 — 2ху + у 2 . Чтобы вычислить квадрат разности двух выражений нужно от квадрата первого выражения отнять удвоенное произведение первого выражения на второе плюс квадрат второго выражения.

Четвертая (х + у) 3 = х 3 + 3х 2 у + 3ху 2 + у 3. Чтобы вычислить куб суммы двух выражений нужно к кубу первого выражения прибавить утроенное произведение квадрата первого выражения на второе плюс утроенное произведение первого выражения на квадрат второго плюс куб второго выражения.

Пятая (х — у) 3 = х 3 — 3х 2 у + 3ху 2 — у 3 . Чтобы рассчитать куб разности двух выражений необходимо от куба первого выражения отнять утроенное произведение квадрата первого выражения на второе плюс утроенное произведение первого выражения на квадрат второго минус куб второго выражения.

Шестая х 3 + у 3 = (х + у) (х 2 — ху + у 2) Чтобы высчитать сумму кубов двух выражений нужно умножить суммы первого и второго выражения на неполный квадрат разности этих выражений.

Седьмая х 3 — у 3 = (х — у) (х 2 + ху + у 2) Чтобы произвести вычисление разности кубов двух выражений надо умножить разность первого и второго выражения на неполный квадрат суммы этих выражений.

Не сложно запомнить, что все формулы применяются для произведения расчетов и в противоположном направлении (справа налево).

О существовании этих закономерностей з нали еще около 4 тысяч лет тому назад. Их широко применяли жители древнего Вавилона и Египта. Но в те эпохи они выражались словесно или геометрически и при расчетах не использовали буквы.

Разберем доказательство квадрата суммы (а + b) 2 = a 2 +2ab +b 2 .

Первым эту математическую закономерность доказал древнегреческий учёный Евклид, работавший в Александрии в III веке до н. э., он использовал для этого геометрический способ доказательства формулы, так как буквами для обозначения чисел не пользовались и учёные древней Эллады. Ими повсеместно употреблялись не “а 2 ”, а “квадрат на отрезке а”, не “ab”, а “прямоугольник , заключенный между отрезками a и b”.

Соблюдение Вашей конфиденциальности важно для нас. По этой причине, мы разработали Политику Конфиденциальности, которая описывает, как мы используем и храним Вашу информацию. Пожалуйста, ознакомьтесь с нашими правилами соблюдения конфиденциальности и сообщите нам, если у вас возникнут какие-либо вопросы.

Сбор и использование персональной информации

Под персональной информацией понимаются данные, которые могут быть использованы для идентификации определенного лица либо связи с ним.

От вас может быть запрошено предоставление вашей персональной информации в любой момент, когда вы связываетесь с нами.

Ниже приведены некоторые примеры типов персональной информации, которую мы можем собирать, и как мы можем использовать такую информацию.

Какую персональную информацию мы собираем:

  • Когда вы оставляете заявку на сайте, мы можем собирать различную информацию, включая ваши имя, номер телефона, адрес электронной почты и т.д.

Как мы используем вашу персональную информацию:

  • Собираемая нами персональная информация позволяет нам связываться с вами и сообщать об уникальных предложениях, акциях и других мероприятиях и ближайших событиях.
  • Время от времени, мы можем использовать вашу персональную информацию для отправки важных уведомлений и сообщений.
  • Мы также можем использовать персональную информацию для внутренних целей, таких как проведения аудита, анализа данных и различных исследований в целях улучшения услуг предоставляемых нами и предоставления Вам рекомендаций относительно наших услуг.
  • Если вы принимаете участие в розыгрыше призов, конкурсе или сходном стимулирующем мероприятии, мы можем использовать предоставляемую вами информацию для управления такими программами.

Раскрытие информации третьим лицам

Мы не раскрываем полученную от Вас информацию третьим лицам.

Исключения:

  • В случае если необходимо — в соответствии с законом, судебным порядком, в судебном разбирательстве, и/или на основании публичных запросов или запросов от государственных органов на территории РФ — раскрыть вашу персональную информацию. Мы также можем раскрывать информацию о вас если мы определим, что такое раскрытие необходимо или уместно в целях безопасности, поддержания правопорядка, или иных общественно важных случаях.
  • В случае реорганизации, слияния или продажи мы можем передать собираемую нами персональную информацию соответствующему третьему лицу – правопреемнику.

Защита персональной информации

Мы предпринимаем меры предосторожности — включая административные, технические и физические — для защиты вашей персональной информации от утраты, кражи, и недобросовестного использования, а также от несанкционированного доступа, раскрытия, изменения и уничтожения.

Соблюдение вашей конфиденциальности на уровне компании

Для того чтобы убедиться, что ваша персональная информация находится в безопасности, мы доводим нормы соблюдения конфиденциальности и безопасности до наших сотрудников, и строго следим за исполнением мер соблюдения конфиденциальности.

Формулы или правила сокращенного умножения используются в арифметике, а точнее — в алгебре, для более быстрого процесса вычисления больших алгебраических выражений. Сами же формулы получены из существующих в алгебре правил для умножения нескольких многочленов.

Использование данных формул обеспечивает достаточно оперативное решение различных математических задач, а также помогает осуществлять упрощение выражений. Правила алгебраических преобразований позволяют выполнять некоторые манипуляции с выражениями, следуя которым можно получить в левой части равенства выражение, стоящее в правой части, или преобразовать правую часть равенства (чтобы получить выражение, стоящее в левой части после знака равенства).

Удобно знать формулы, применяемые для сокращенного умножения, на память, так как они нередко используются при решении задач и уравнений. Ниже перечислены основные формулы, входящие в данный список, и их наименование.

Квадрат суммы

Чтобы вычислить квадрат суммы, необходимо найти сумму, состоящую из квадрата первого слагаемого, удвоенного произведения первого слагаемого на второе и квадрата второго. В виде выражения данное правило записывается следующим образом: (а + с)² = a² + 2ас + с².

Квадрат разности

Чтобы вычислить квадрат разности, необходимо вычислить сумму, состоящую из квадрата первого числа, удвоенного произведения первого числа на второе (взятое с противоположным знаком) и квадрата второго числа. В виде выражения данное правило выглядит следующим образом: (а — с)² = а² — 2ас + с².

Разность квадратов

Формула разности двух чисел, возведенных в квадрат, равна произведению суммы этих чисел на их разность. В виде выражения данное правило выглядит следующим образом: a² — с² = (a + с)·(a — с).

Куб суммы

Чтобы вычислить куб суммы двух слагаемых, необходимо вычислить сумму, состоящую из куба первого слагаемого, утроенного произведения квадрата первого слагаемого и второго, утроенного произведения первого слагаемого и второго в квадрате, а также куба второго слагаемого. В виде выражения данное правило выглядит следующим образом: (а + с)³ = а³ + 3а²с + 3ас² + с³.

Сумма кубов

Согласно формуле, приравнивается к произведению суммы данных слагаемых на их неполный квадрат разности. В виде выражения данное правило выглядит следующим образом: а³ + с³ = (а + с)·(а² — ас + с²).

Пример. Необходимо вычислить объем фигуры, которая образована сложением двух кубов. Известны лишь величины их сторон.

Если значения сторон небольшие, то выполнить вычисления просто.

Если же длины сторон выражаются в громоздких числах, то в этом случае проще применить формулу «Сумма кубов», которая значительно упростит вычисления.

Куб разности

Выражение для кубической разности звучит так: как сумма третьей степени первого члена, утроенного отрицательного произведения квадрата первого члена на второй, утроенного произведения первого члена на квадрат второго и отрицательного куба второго члена. В виде математического выражения куб разности выглядит следующим образом: (а — с)³ = а³ — 3а²с + 3ас² — с³.

Разность кубов

Формула разности кубов отличается от суммы кубов лишь одним знаком. Таким образом, разность кубов — формула, равная произведению разности данных чисел на их неполный квадрат суммы. В виде математического выражения разность кубов выглядит следующим образом: а 3 — с 3 = (а — с)(а 2 + ас + с 2).

Пример. Необходимо вычислить объем фигуры, которая останется после вычитания из объема синего куба объемной фигуры желтого цвета, которая также является кубом. Известна лишь величина стороны маленького и большого куба.

Если значения сторон небольшие, то вычисления довольно просты. А если длины сторон выражаются в значительных числах, то стоит применить формулу, озаглавленную «Разность кубов» (или «Куб разности»), которае значительно упростит вычисления.

В предыдущем уроке мы разобрались с разложением на множители. Освоили два способа: вынесение общего множителя за скобки и группировку. В этом уроке — следующий мощный способ: формулы сокращённого умножения . В краткой записи — ФСУ.

Формулы сокращённого умножения (квадрат суммы и разности, куб суммы и разности, разность квадратов, сумма и разность кубов) крайне необходимы во всех разделах математики. Они применяются в упрощении выражений, решении уравнений, умножении многочленов, сокращении дробей, решении интегралов и т.д. и т.п. Короче, есть все основания разобраться с ними. Понять откуда они берутся, зачем они нужны, как их запомнить и как применять.

Разбираемся?)

Откуда берутся формулы сокращённого умножения?

Равенства 6 и 7 записаны не очень привычно. Как бы наоборот. Это специально.) Любое равенство работает как слева направо, так и справа налево. В такой записи понятнее, откуда берутся ФСУ.

Они берутся из умножения.) Например:

(a+b) 2 =(a+b)(a+b)=a 2 +ab+ba+b 2 =a 2 +2ab+b 2

Вот и всё, никаких научных хитростей. Просто перемножаем скобки и приводим подобные. Так получаются все формулы сокращённого умножения. Сокращённое умножение — это потому, что в самих формулах нет перемножения скобок и приведения подобных. Сокращены.) Сразу дан результат.

ФСУ нужно знать наизусть. Без первых трёх можно не мечтать о тройке, без остальных — о четвёрке с пятёркой.)

Зачем нужны формулы сокращённого умножения?

Есть две причины, выучить, даже зазубрить эти формулы. Первая — готовый ответ на автомате резко уменьшает количество ошибок. Но это не самая главная причина. А вот вторая…

Кстати, у меня есть ещё парочка интересных сайтов для Вас.)

Можно потренироваться в решении примеров и узнать свой уровень. Тестирование с мгновенной проверкой. Учимся — с интересом!)

можно познакомиться с функциями и производными.

При расчёте алгебраических многочленов для упрощения вычислений используются формулы сокращенного умножения . Всего таких формул семь. Их все необходимо знать наизусть.

Следует также помнить, что вместо a и b в формулах могут стоять как числа, так и любые другие алгебраические многочлены.

Разность квадратов

Разность квадратов двух чисел равна произведению разности этих чисел и их суммы.

a 2 — b 2 = (a — b)(a + b)

Квадрат суммы

Квадрат суммы двух чисел равен квадрату первого числа плюс удвоенное произведение первого числа на второе плюс квадрат второго числа.

(a + b) 2 = a 2 + 2ab + b 2

Обратите внимание, что с помощью этой формулы сокращённого умножения легко находить квадраты больших чисел , не используя калькулятор или умножение в столбик. Поясним на примере:

Найти 112 2 .

Разложим 112 на сумму чисел, чьи квадраты мы хорошо помним.2
112 = 100 + 1

Запишем сумму чисел в скобки и поставим над скобками квадрат.
112 2 = (100 + 12) 2

Воспользуемся формулой квадрата суммы:
112 2 = (100 + 12) 2 = 100 2 + 2 x 100 x 12 + 12 2 = 10 000 + 2 400 + 144 = 12 544

Помните, что формула квадрат суммы также справедлива для любых алгебраических многочленов.

(8a + с) 2 = 64a 2 + 16ac + c 2

Предостережение!!!

(a + b) 2 не равно a 2 + b 2

Квадрат разности

Квадрат разности двух чисел равен квадрату первого числа минус удвоенное произведение первого на второе плюс квадрат второго числа.

(a — b) 2 = a 2 — 2ab + b 2

Также стоит запомнить весьма полезное преобразование:

(a — b) 2 = (b — a) 2
Формула выше доказывается простым раскрытием скобок:

(a — b) 2 = a 2 — 2ab + b 2 = b 2 — 2ab + a 2 = (b — a) 2

Куб суммы

Куб суммы двух чисел равен кубу первого числа плюс утроенное произведение квадрата первого числа на второе плюс утроенное произведение первого на квадрат второго плюс куб второго.

(a + b) 3 = a 3 + 3a 2 b + 3ab 2 + b 3

Запомнить эту «страшную» на вид формулу довольно просто.

Выучите, что в начале идёт a 3 .

Два многочлена посередине имеют коэффициенты 3.

В спомним, что любое число в нулевой степени есть 1. (a 0 = 1, b 0 = 1). Легко заметить, что в формуле идёт понижение степени a и увеличение степени b. В этом можно убедиться:
(a + b) 3 = a 3 b 0 + 3a 2 b 1 + 3a 1 b 2 + b 3 a 0 = a 3 + 3a 2 b + 3ab 2 + b 3

Предостережение!!!

(a + b) 3 не равно a 3 + b 3

Куб разности

Куб разности двух чисел равен кубу первого числа минус утроенное произведение квадрата первого числа на второе плюс утроенное произведение первого числа на квадрат второго минус куб второго.

(a — b) 3 = a 3 — 3a 2 b + 3ab 2 — b 3

Запоминается эта формула как и предыдущая, но только с учётом чередования знаков «+» и «-». Перед первым членом a 3 стоит «+» (по правилам математики мы его не пишем). Значит, перед следующим членом будет стоять «-», затем опять «+» и т.д.

(a — b) 3 = + a 3 — 3a 2 b + 3ab 2 — b 3 = a 3 — 3a 2 b + 3ab 2 — b 3

Сумма кубов ( Не путать с кубом суммы!)

Сумма кубов равна произведению суммы двух чисел на неполный квадрат разности.

a 3 + b 3 = (a + b)(a 2 — ab + b 2)

Сумма кубов — это произведение двух скобок.

Первая скобка — сумма двух чисел.

Вторая скобка — неполный квадрат разности чисел. Неполным квадратом разности называют выражение:

A 2 — ab + b 2
Данный квадрат неполный, так как посередине вместо удвоенного произведения обычное произведение чисел.

Разность кубов (Не путать с кубом разности!!!)

Разность кубов равна произведению разности двух чисел на неполный квадрат суммы.

a 3 — b 3 = (a — b)(a 2 + ab + b 2)

Будьте внимательны при записи знаков. Следует помнить, что все формулы, приведённые выше, используется также и справа налево.

Трудно запоминаются формулы сокращенного умножения? Делу легко помочь. Нужно просто запомнить, как изображается такая простая вещь, как треугольник Паскаля. Тогда вы вспомните эти формулы всегда и везде, вернее, не вспомните, а восстановите.

Что же такое треугольник Паскаля? Этот треугольник состоит из коэффициентов, которые входят в разложение любой степени двучлена вида в многочлен.

Разложим, например, :

В этой записи легко запоминается, что вначале стоит куб первого, а в конце — куб второго числа. А вот что посередине — запоминается сложно. И даже то, что в каждом следующем слагаемом степень одного множителя все время уменьшается, а второго — увеличивается — несложно заметить и запомнить, труднее дело обстоит с запоминанием коэффициентов и знаков (плюс там или минус?).

Итак, сначала коэффициенты. Не надо их запоминать! На полях тетрадки быстренько рисуем треугольник Паскаля, и вот они — коэффициенты, уже перед нами. Рисовать начинаем с трех единичек, одна сверху, две ниже, правее и левее — ага, уже треугольник получается:

Первая строка, с одной единичкой — нулевая. Потом идет первая, вторая, третья и так далее. Чтобы получить вторую строку, нужно по краям снова приписать единички, а в центре записать число, полученное сложением двух чисел, стоящих над ним:

Записываем третью строку: опять по краям единицы, и опять, чтобы получить следующее число в новой строке, сложим числа, стоящие над ним в предыдущей:


Как вы уже догадались, мы получаем в каждой строке коэффициенты из разложения двучлена в многочлен:


Ну а знаки запомнить еще проще: первый — такой же, как в раскладываемом двучлене (раскладываем сумму — значит, плюс, разность — значит, минус), а дальше знаки чередуются!

Вот такая это полезная штука — треугольник Паскаля. 2\right)\]

В предыдущих уроках мы рассмотрели два способа разложения многочлена на множители: вынесение общего множителя за скобки и способ группировки .

В этом уроке мы рассмотрим еще один способ разложения многочлена на множители с применением формул сокращённого умножения .

Рекомендуем каждую формулу прописать не менее 12 раз. Для лучшего запоминания выпишите все формулы сокращённого умножения себе на небольшую шпаргалку .

Вспомним, как выглядит формула разности кубов.

a 3 − b 3 = (a − b)(a 2 + ab + b 2)

Формула разности кубов не очень проста для запоминания, поэтому рекомендуем использовать специальный способ для её запоминания.

Важно понимать, что любая формула сокращённого умножения действует и в обратную сторону .

(a − b)(a 2 + ab + b 2) = a 3 − b 3

Рассмотрим пример. Необходимо разложить на множители разность кубов.

Обратим внимание, что «27а 3 » — это «(3а) 3 », значит, для формулы разности кубов вместо «a » мы используем «3a ».

Используем формулу разности кубов. На месте «a 3 » у нас стоит «27a 3 », а на месте «b 3 », как и в формуле, стоит «b 3 ».

Применение разности кубов в обратную сторону

Рассмотрим другой пример. Требуется преобразовать произведение многочленов в разность кубов, используя формулу сокращенного умножения.

Обратите внимание, что произведение многочленов «(x − 1)(x 2 + x + 1) » напоминает правую часть формулы разности кубов «», только вместо «a » стоит «x », а на месте «b » стоит «1 ».

Используем для «(x − 1)(x 2 + x + 1) » формулу разности кубов в обратную сторону.


Рассмотрим пример сложнее. Требуется упростить произведение многочленов.

Если сравнить «(y 2 − 1)(y 4 + y 2 + 1) » с правой частью формулы разности кубов
«a 3 − b 3 = (a − b)(a 2 + ab + b 2) », то можно понять, что на месте «a » из первой скобки стоит «y 2 , а на месте «b » стоит «1 ».

Формулы или правила сокращенного умножения используются в арифметике, а точнее — в алгебре, для более быстрого процесса вычисления больших алгебраических выражений. Сами же формулы получены из существующих в алгебре правил для умножения нескольких многочленов.

Использование данных формул обеспечивает достаточно оперативное решение различных математических задач, а также помогает осуществлять упрощение выражений. Правила алгебраических преобразований позволяют выполнять некоторые манипуляции с выражениями, следуя которым можно получить в левой части равенства выражение, стоящее в правой части, или преобразовать правую часть равенства (чтобы получить выражение, стоящее в левой части после знака равенства).

Удобно знать формулы, применяемые для сокращенного умножения, на память, так как они нередко используются при решении задач и уравнений. Ниже перечислены основные формулы, входящие в данный список, и их наименование.

Квадрат суммы

Чтобы вычислить квадрат суммы, необходимо найти сумму, состоящую из квадрата первого слагаемого, удвоенного произведения первого слагаемого на второе и квадрата второго. В виде выражения данное правило записывается следующим образом: (а + с)² = a² + 2ас + с².

Квадрат разности

Чтобы вычислить квадрат разности, необходимо вычислить сумму, состоящую из квадрата первого числа, удвоенного произведения первого числа на второе (взятое с противоположным знаком) и квадрата второго числа. В виде выражения данное правило выглядит следующим образом: (а — с)² = а² — 2ас + с².

Разность квадратов

Формула разности двух чисел, возведенных в квадрат, равна произведению суммы этих чисел на их разность. В виде выражения данное правило выглядит следующим образом: a² — с² = (a + с)·(a — с).

Куб суммы

Чтобы вычислить куб суммы двух слагаемых, необходимо вычислить сумму, состоящую из куба первого слагаемого, утроенного произведения квадрата первого слагаемого и второго, утроенного произведения первого слагаемого и второго в квадрате, а также куба второго слагаемого. В виде выражения данное правило выглядит следующим образом: (а + с)³ = а³ + 3а²с + 3ас² + с³.

Сумма кубов

Согласно формуле, приравнивается к произведению суммы данных слагаемых на их неполный квадрат разности. В виде выражения данное правило выглядит следующим образом: а³ + с³ = (а + с)·(а² — ас + с²).

Пример. Необходимо вычислить объем фигуры, которая образована сложением двух кубов. Известны лишь величины их сторон.

Если значения сторон небольшие, то выполнить вычисления просто.

Если же длины сторон выражаются в громоздких числах, то в этом случае проще применить формулу «Сумма кубов», которая значительно упростит вычисления.

Куб разности

Выражение для кубической разности звучит так: как сумма третьей степени первого члена, утроенного отрицательного произведения квадрата первого члена на второй, утроенного произведения первого члена на квадрат второго и отрицательного куба второго члена. В виде математического выражения куб разности выглядит следующим образом: (а — с)³ = а³ — 3а²с + 3ас² — с³.

Разность кубов

Формула разности кубов отличается от суммы кубов лишь одним знаком. Таким образом, разность кубов — формула, равная произведению разности данных чисел на их неполный квадрат суммы. В виде разность кубов выглядит следующим образом: а 3 — с 3 = (а — с)(а 2 + ас + с 2).

Пример. Необходимо вычислить объем фигуры, которая останется после вычитания из объема синего куба объемной фигуры желтого цвета, которая также является кубом. Известна лишь величина стороны маленького и большого куба.

Если значения сторон небольшие, то вычисления довольно просты. А если длины сторон выражаются в значительных числах, то стоит применить формулу, озаглавленную «Разность кубов» (или «Куб разности»), которае значительно упростит вычисления.

Формулы сокращенного умножения.

Изучение формул сокращенного умножения: квадрата суммы и квадрата разности двух выражений; разности квадратов двух выражений; куба суммы и куба разности двух выражений; суммы и разности кубов двух выражений.

Применение формул сокращенного умножения при решении примеров.

Для упрощения выражений, разложения многочленов на множители, приведения многочленов к стандартному виду используются формулы сокращенного умножения. Формулы сокращенного умножения нужно знать наизусть .

Пусть а, b R. Тогда:

1. Квадрат суммы двух выражений равен квадрату первого выражения плюс удвоенное произведение первого выражения на второе плюс квадрат второго выражения.

(a + b) 2 = a 2 + 2ab + b 2

2. Квадрат разности двух выражений равен квадрату первого выражения минус удвоенное произведение первого выражения на второе плюс квадрат второго выражения.

(a — b) 2 = a 2 — 2ab + b 2

3. Разность квадратов двух выражений равна произведению разности этих выражений и их суммы.

a 2 — b 2 = (a -b) (a+b)

4. Куб суммы двух выражений равен кубу первого выражения плюс утроенное произведение квадрата первого выражения на второе плюс утроенное произведение первого выражения на квадрат второго плюс куб второго выражения.

(a + b) 3 = a 3 + 3a 2 b + 3ab 2 + b 3

5. Куб разности двух выражений равен кубу первого выражения минус утроенное произведение квадрата первого выражения на второе плюс утроенное произведение первого выражения на квадрат второго минус куб второго выражения.

(a — b) 3 = a 3 — 3a 2 b + 3ab 2 — b 3

6. Сумма кубов двух выражений равна произведению суммы первого и второго выражения на неполный квадрат разности этих выражений.

a 3 + b 3 = (a + b) (a 2 — ab + b 2)

7. Разность кубов двух выражений равна произведению разности первого и второго выражения на неполный квадрат суммы этих выражений.

a 3 — b 3 = (a — b) (a 2 + ab + b 2)

Применение формул сокращенного умножения при решении примеров.

Пример 1.

Вычислить

а) Используя формулу квадрата суммы двух выражений, имеем

(40+1) 2 = 40 2 + 2 · 40 · 1 + 1 2 = 1600 + 80 + 1 = 1681

б) Используя формулу квадрата разности двух выражений, получим

98 2 = (100 – 2) 2 = 100 2 — 2 · 100 · 2 + 2 2 = 10000 – 400 + 4 = 9604

Пример 2.

Вычислить

Используя формулу разности квадратов двух выражений, получим

Пример 3.

Упростить выражение

(х — у) 2 + (х + у) 2

Воспользуемся формулами квадрата суммы и квадрата разности двух выражений

(х — у) 2 + (х + у) 2 = х 2 — 2ху + у 2 + х 2 + 2ху + у 2 = 2х 2 + 2у 2

Формулы сокращенного умножения в одной таблице:

(a + b) 2 = a 2 + 2ab + b 2
(a — b) 2 = a 2 — 2ab + b 2
a 2 — b 2 = (a — b) (a+b)
(a + b) 3 = a 3 + 3a 2 b + 3ab 2 + b 3
(a — b) 3 = a 3 — 3a 2 b + 3ab 2 — b 3
a 3 + b 3 = (a + b) (a 2 — ab + b 2)
a 3 — b 3 = (a — b) (a 2 + ab + b 2)

Х 1 в кубе формула.

Формулы сокращенного умножения

Формулы сокращенного умножения.

Изучение формул сокращенного умножения: квадрата суммы и квадрата разности двух выражений; разности квадратов двух выражений; куба суммы и куба разности двух выражений; суммы и разности кубов двух выражений.

Применение формул сокращенного умножения при решении примеров.

Для упрощения выражений, разложения многочленов на множители, приведения многочленов к стандартному виду используются формулы сокращенного умножения. Формулы сокращенного умножения нужно знать наизусть .

Пусть а, b R. Тогда:

1. Квадрат суммы двух выражений равен квадрату первого выражения плюс удвоенное произведение первого выражения на второе плюс квадрат второго выражения.

(a + b) 2 = a 2 + 2ab + b 2

2. Квадрат разности двух выражений равен квадрату первого выражения минус удвоенное произведение первого выражения на второе плюс квадрат второго выражения.

(a — b) 2 = a 2 — 2ab + b 2

3. Разность квадратов двух выражений равна произведению разности этих выражений и их суммы.

a 2 — b 2 = (a -b) (a+b)

4. Куб суммы двух выражений равен кубу первого выражения плюс утроенное произведение квадрата первого выражения на второе плюс утроенное произведение первого выражения на квадрат второго плюс куб второго выражения.

(a + b) 3 = a 3 + 3a 2 b + 3ab 2 + b 3

5. Куб разности двух выражений равен кубу первого выражения минус утроенное произведение квадрата первого выражения на второе плюс утроенное произведение первого выражения на квадрат второго минус куб второго выражения.

(a — b) 3 = a 3 — 3a 2 b + 3ab 2 — b 3

6. Сумма кубов двух выражений равна произведению суммы первого и второго выражения на неполный квадрат разности этих выражений.

a 3 + b 3 = (a + b) (a 2 — ab + b 2)

7. Разность кубов двух выражений равна произведению разности первого и второго выражения на неполный квадрат суммы этих выражений.

a 3 — b 3 = (a — b) (a 2 + ab + b 2)

Применение формул сокращенного умножения при решении примеров.

Пример 1.

Вычислить

а) Используя формулу квадрата суммы двух выражений, имеем

(40+1) 2 = 40 2 + 2 · 40 · 1 + 1 2 = 1600 + 80 + 1 = 1681

б) Используя формулу квадрата разности двух выражений, получим

98 2 = (100 – 2) 2 = 100 2 — 2 · 100 · 2 + 2 2 = 10000 – 400 + 4 = 9604

Пример 2.

Вычислить

Используя формулу разности квадратов двух выражений, получим

Пример 3.

Упростить выражение

(х — у) 2 + (х + у) 2

Воспользуемся формулами квадрата суммы и квадрата разности двух выражений

(х — у) 2 + (х + у) 2 = х 2 — 2ху + у 2 + х 2 + 2ху + у 2 = 2х 2 + 2у 2

Формулы сокращенного умножения в одной таблице:

(a + b) 2 = a 2 + 2ab + b 2
(a — b) 2 = a 2 — 2ab + b 2
a 2 — b 2 = (a — b) (a+b)
(a + b) 3 = a 3 + 3a 2 b + 3ab 2 + b 3
(a — b) 3 = a 3 — 3a 2 b + 3ab 2 — b 3
a 3 + b 3 = (a + b) (a 2 — ab + b 2)
a 3 — b 3 = (a — b) (a 2 + ab + b 2)

Математические выражения (формулы) сокращённого умножения (квадрат суммы и разности, куб суммы и разности, разность квадратов, сумма и разность кубов) крайне не заменимы во многих областях точных наук. Эти 7 символьных записей не заменимы при упрощении выражений, решении уравнений, при умножении многочленов, сокращении дробей , решении интегралов и многом другом. А значит будет очень полезно разобраться как они получаются, для чего они нужны, и самое главное, как их запомнить и потом применять. Потом применяя формулы сокращенного умножения на практике самым сложным будет увидеть, что есть х и что есть у. Очевидно, что никаких ограничений для a и b нет, а значит это могут быть любые числовые или буквенные выражения.

И так вот они:

Первая х 2 — у 2 = (х — у) (х+у) .Чтобы рассчитать разность квадратов двух выражений надо перемножить разности этих выражений на их суммы.

Вторая (х + у) 2 = х 2 + 2ху + у 2 . Чтобы найти квадрат суммы двух выражений нужно к квадрату первого выражения прибавить удвоенное произведение первого выражения на второе плюс квадрат второго выражения.

Третья (х — у) 2 = х 2 — 2ху + у 2 . Чтобы вычислить квадрат разности двух выражений нужно от квадрата первого выражения отнять удвоенное произведение первого выражения на второе плюс квадрат второго выражения.

Четвертая (х + у) 3 = х 3 + 3х 2 у + 3ху 2 + у 3. Чтобы вычислить куб суммы двух выражений нужно к кубу первого выражения прибавить утроенное произведение квадрата первого выражения на второе плюс утроенное произведение первого выражения на квадрат второго плюс куб второго выражения.

Пятая (х — у) 3 = х 3 — 3х 2 у + 3ху 2 — у 3 . Чтобы рассчитать куб разности двух выражений необходимо от куба первого выражения отнять утроенное произведение квадрата первого выражения на второе плюс утроенное произведение первого выражения на квадрат второго минус куб второго выражения.

Шестая х 3 + у 3 = (х + у) (х 2 — ху + у 2) Чтобы высчитать сумму кубов двух выражений нужно умножить суммы первого и второго выражения на неполный квадрат разности этих выражений.

Седьмая х 3 — у 3 = (х — у) (х 2 + ху + у 2) Чтобы произвести вычисление разности кубов двух выражений надо умножить разность первого и второго выражения на неполный квадрат суммы этих выражений.

Не сложно запомнить, что все формулы применяются для произведения расчетов и в противоположном направлении (справа налево).

О существовании этих закономерностей з нали еще около 4 тысяч лет тому назад. Их широко применяли жители древнего Вавилона и Египта. Но в те эпохи они выражались словесно или геометрически и при расчетах не использовали буквы.

Разберем доказательство квадрата суммы (а + b) 2 = a 2 +2ab +b 2 .

Первым эту математическую закономерность доказал древнегреческий учёный Евклид, работавший в Александрии в III веке до н.э., он использовал для этого геометрический способ доказательства формулы, так как буквами для обозначения чисел не пользовались и учёные древней Эллады. Ими повсеместно употреблялись не “а 2 ”, а “квадрат на отрезке а”, не “ab”, а “прямоугольник , заключенный между отрезками a и b”.

Информация о марке, модели и альтернативных названиях конкретного устройства, если таковые имеются.

Дизайн

Информация о размерах и весе устройства, представленная в разных единицах измерения. Использованные материалы, предлагаемые цвета, сертификаты.

SIM-карта

SIM-карта используется в мобильных устройствах для сохранения данных, удостоверяющих аутентичность абонентов мобильных услуг.

Мобильные сети

Мобильная сеть — это радио-система, которая позволяет множеству мобильных устройств обмениваться данными между собой.

Технологии мобильной связи и скорость передачи данных

Коммуникация между устройствами в мобильных сетях осуществляется посредством технологий, предоставляющих разные скорости передачи данных.

Oперационная система

Операционная система — это системное программное обеспечение, управляющее и координирующее работу хардверных компонентов в устройстве.

SoC (Система на кристалле)

Система на кристалле (SoC) включает в один чип все самые главные хардверные компоненты мобильного устройства.

SoC (Система на кристалле)

Система на кристалле (SoC) интегрирует различные хардверные компоненты, таких как процессор, графический процессор, память, периферия, интерфейсы и др., а также и софтвер, необходимый для их функционирования.

MediaTek Helio X20 (MT6797)
Технологический процесс

Информация о технологическом процессе, по которому изготовлен чип. Величиной в нанометрах измеряют половину расстояния между элементами в процессоре.

20 нм (нанометры)
Процессор (CPU)

Основная функция процессора (CPU) мобильного устройства — это интерпретация и выполнение инструкций, содержащихся в программных приложениях.

2x 2.3 GHz ARM Cortex-A72, 4x 1.85 GHz ARM Cortex-A53, 4x 1.4 GHz ARM Cortex-A53
Разрядность процессора

Разрядность (биты) процессора определяется размером (в битах) регистров, адресных шин и шин для данных. 64-битные процессоры обладают более высокой производительностью по сравнению с 32-битными, которые со своей стороны более производительны, чем 16-битные процессоры.

64 бит
Архитектура набора команд

Инструкции — это команды, с помощью которых софтуер задает/управляет работой процессора. Информация об наборе командов (ISA), которые процессор может выполнять.

ARMv8-A
Kоличество ядер процессора

Ядро процессора выполняет программные инструкции. Существуют процессоры с одним, двумя и более ядрами. Наличие большего количества ядер увеличивает производительность, позволяя параллельное выполнение множества инструкций.

10
Тактовая частота процессора

Тактовая частота процессора описывает его скорость посредством циклов в секунду. Она измеряется в мегагерцах (MHz) или гигагерцах (GHz).

2300 МГц (мегагерцы)
Графический процессор (GPU)

Графический процессор (GPU) обрабатывает вычисления для различных 2D/3D графических приложений. В мобильных устройствах он используется чаще всего играми, потребительским интерфейсом, видео-приложениями и др.

ARM Mali-T880 MP4
Kоличество ядер графического процессора

Подобно процессору, графический процессор состоит из нескольких рабочих частей, которые называются ядрами. Они обрабатывают графические вычисления разных приложений.

4
Тактовая частота графического процессора

Скорость работы — это тактовая частота графического процессора, которая измеряется в мегагерцах (MHz) или гигагерцах (GHz).

780 МГц (мегагерцы)
Объём оперативной памяти (RAM)

Оперативная память (RAM) используется операционной системой и всеми инсталлированными приложениями. Данные, которые сохраняются в оперативной памяти, теряются после выключения или рестартирования устройства.

4 ГБ (гигабайты)
Тип оперативной памяти (RAM)

Информация о типе оперативной памяти (RAM) используемый устройством.

LPDDR3
Количество каналов оперативной памяти

Информация о количестве каналов оперативной памяти каторые интегрированы в SoC. Больше каналов означает более высокие скорости передачи данных.

Двухканальная
Частота оперативной памяти

Частота оперативной памяти определяет ее скорость работы, более конкретно, скорость чтения/записи данных.

800 МГц (мегагерцы)

Встроенная память

Каждое мобильное устройство имеет встроенную (несъемную) память с фиксированным объемом.

Карты памяти

Карты памяти используются в мобильных устройствах для увеличения объема памяти для сохранения данных.

Экран

Экран мобильного устройства характеризуется своей технологией, разрешением, плотностью пикселей, длиной диагонали, глубиной цвета и др.

Датчики

Различные датчики выполняют различные количественные измерения и конвертируют физические показатели в сигналы, которые распознает мобильное устройство.

Тыловая камера

Основная камера мобильного устройства обычно расположена на его задней панели и может сочетаться с одной или несколькими дополнительными камерами.

Фронтальная камера

Смартфоны имеют одну или несколько фронтальных камер различного дизайна — pop-up камера, поворотная камера, вырез или дырка в дисплее, камера под дисплеем.

Аудио

Информация о типе громкоговорителей и поддерживаемых устройством аудиотехнологиях.

Радио

Радио мобильного устройства представляет собой встроенный FM-приемник.

Определение местоположения

Информация о технологиях навигации и определения местоположения, поддерживаемых устройством.

Wi-Fi

Wi-Fi — это технология, которая обеспечивает беспроводную связь для передачи данных на близкие расстояния между различными устройствами.

Bluetooth

Bluetooth — это стандарт безопасного беспроводного переноса данных между различными устройствами разного типа на небольшие расстояния.

USB

USB (Universal Serial Bus) — это индустриальный стандарт, который позволяет разным электронным устройствам обмениваться данными.

Разъём для наушников

Это аудиоконнектор, который называется еще и аудиоразъемом.

Наиболее широко используемый стандарт в мобильных устройствах — это 3.5 мм разъем для наушников.

Подключение устройств

Информация о других важных технологиях подключения, поддерживаемых устройством.

Браузер

Веб-браузер — это программное приложение для доступа и рассматривания информации в интернете.

Форматы/кодеки видео файлов

Мобильные устройства поддерживают разные форматы и кодеки видео файлов, которые соответственно сохраняют и кодируют/декодируют цифровые видеоданные.

Аккумулятор

Аккумуляторы мобильных устройств отличаются друг от друга по своей емкости и технологии. Они обеспечивают электрический заряд, необходимый для их функционирования.

Wi-Fi

3 формула используется для вычисления куба двучлена. Формула также известна как куб разницы между двумя терминами. Чтобы найти формулу (a – b) 3 , мы просто умножим (a – b) (a – b) (a – b).

(a — b) 3  = (a — b)(a — b)(a — b)

 = (a 2  — 2ab + b 2 )(a — b)

 = а 3  — а 2 б — 2а 2 б + 2аб 2  + аб 2  — б 3  

2 = а 0009 3 -3A 2 B + 3AB 2 -B 3

= A 3 -3AB (A-B)-B 3

Следовательно, (A-B) 3 Формула-это :

(a — b) 3  = a 3  — 3a 2 b + 3ab 2  — b 3

9000-е секунды.

Воспользуйтесь нашим бесплатным онлайн-калькулятором, чтобы решить сложные вопросы. С Cuemath находите решения простыми и легкими шагами. 93 Формула

Пример 1:  Решите следующее выражение, используя (a — b) 3  формула:
(2x — 3y) 3

Решение:      

Найти: (2x — 3y) 3
Используя (a – b) 3 Формула,
(a — b) 3  = a 3  — 3a 2 b + 3ab 2  — b 3
= (2x) 3  — 3 × (2x) 2  × 3y + 3 × (2x) × (3y) 2  — (3y) 3
= 8x 3 -36x 2 y + 54xy 2 -27y 3

Ответ: (2x-3y) 3 = 8x 3 -36x 2 Y + 54xy 2 — 27y 3

Пример 2:  Найдите значение x 3  — y 3  если x — y = 5 и xyula = 2, используя (a — b) 0 3 .

Решение:      

Найти: x 3  — y 3
Дано:
х — у = 5
ху = 2 
Используя (a – b) 3 Формула,
(a — b) 3  = a 3  — 3a 2 b + 3ab 2  — b 3
Здесь а = х; б = у
Следовательно,
(x — y) 3  = x 3  — 3 × x 2  × y   + 3 × x × y 2  — y 3 0 8   (x — y) 3  = x 3  — 3x 2 у + 3xy 2  — y 3  
5 3 = х 3  — 3xy(x — y) — y 3
125 = x 3  — 3 × 2 × 5 — y 3
x 3 -Y 3 = 95

Ответ: x 3 -Y 3 = 95

Пример 3: Решите следующее выражение, используя (A-B) 3 Формула:

(5x — 2y) 3

Решение:      

Найти: (5x — 2y) 3
Используя (a – b) 3 Формула,
(a — b) 3  = a 3  — 3a 2 b + 3ab 2  — b 3
= (5x) 3  — 3 × (5x) 2  × 2y + 3 × (5x) × (2y) 2  — (2y) 3
= 125x 3 -150x 2 Y + 60xy 2 -8y 3

Ответ: (5x-2y) 3 = 125x 3 -150x 93 Формула

Что такое расширение (a — b)

3  Формула?

(a — b) 3  формула читается как минус b целого куба. Его разложение выражается как (a — b) 3  = a 3  — 3a 2 b + 3ab 2  — b 3

Что такое формула 10 900b) 3 ?

Формула (a — b) 3 также известна как одно из важных алгебраических тождеств. Читается как минус b целого куба. Его (а — б) 3 Формула выражена как (A -B) 3 = A 3 -3A 2 B + 3AB 2 -B 3 Как упростить числа, используя (A -B) 3 Формула?

Давайте разберемся в использовании формулы (a — b) 3  с помощью следующего примера.
Пример:  Найдите значение (20 – 5) 3 , используя формулу (a – b) 3  .
Чтобы найти: (20 — 5) 3
Предположим, что a = 20 и b = 5,
Мы подставим их в формулу (a — b) 3 .
(a — b) 3  = a 3  — 3a 2 b + 3ab 2  — b 3
(20-5) 3  = 20 3 — 3(20) 2 (5) + 3(20)(5) 2  — 5 3
= 8000 — 6000 + 1500 — 125
= 3375
Ответ:  (20 — 5) 3  = 3375.

Как использовать формулу (a — b)

3  ?

как власть или нет.

  • Запишите формулу (a — b) 3
  • (a — b) 3  = a 3  — 3a 2 b + 3ab 2  — b 3
  • Подставьте значения a и b в (a — b) 3 формулу и упростите.
  • a Куб минус b Формула куба

    A КУБ МИНУС B КУБ ФОРМУЛА

    В этом разделе мы увидим формулу для

    a — b 3

    Мы уже знаем формулу/разложение для (a — b) 3 .

    , то есть

    (a — b) 3 = A 3 — B 3 — 3AB (A — B)

    Случай 1:

    (A — B) 3 =. a 3  — b 3  — 3ab(a-b)

    Добавьте 3ab(a-b) с каждой стороны.

    (a — b) 3 + 3ab(a — b)  =  a 3  — b 3

    Следовательно, формула для ( 3 — B 3 ) составляет

    A 3 — B 3 = (A — B) 3 + 3AB (A — B)

    Случай 2:

    Из случая 1,

    A 3 — B 3 = (A — B) 3 + 3AB (A — B)

    A 3 — B 3 = (a — b)[(a — b) 2  + 3ab]

    a 3  — b 3   =  (a — b)[a 2  — 2ab + b + 3ab]

    a 3  — b 3   =  (a — b)(a 2  + ab + b 2) 3 — B 3 )-

    A 3 — B 3 = (A- B) (A 2 + AB + B 2 )

    SO,

    ( a — b) и (a 2  + ab + b 2 )

    являются множителями (a 3  — b 3 ).

    Примечание: 

    В зависимости от наших потребностей мы можем использовать формулу либо в случае 1, либо в случае 2 для (a 3  — b 3 ).

    Практические вопросы


    Вопрос 1:

    Фактор:

    x 3 — 1

    Решение:

    Напишите (x 3 — 1).  — б 3 ).

    x 3  — 1 =  x 3  — 1 3

    (x 3  — 1 3 )    имеет форму (a 3 — b 3 ).

    Сравнение (A 3 — B 3 ) и (x 3 — 1 3 ), мы получаем

    A = x

    B = 1

    Записать формулу ( — b 3 ), указанный в случае 2 выше.

    а 3  — b 3   =  (a — b)(a 2  + ab + b 2 )

    Подставьте x вместо a и 1 вместо b.

    x 3  — 1 3   =  (x — 2)(x 2  + x(1) + 1 2 )

    x 3 + = (9 x  0) 2 + x + 1)

    Вопрос 2:

    Фактор:

    8x 3 — 27y 3

    Решение:

    Запись (8x 3 -7999 31010101010111011101110111011101110111011101110111011101110 100001101101110111011101111111110. 29003

    . в виде (а 3  — б 3 ).

    8x 3 — 27y 3 = (2x) 3 — (3Y) 3

    (2x) 3 — (3Y) 3 находится в форме (3Y) 3 находится в форме (3Y) а 3 — б 3 ).

    Сравнение (A 3 — B 3 ) и (2x) 3 — (3Y) 3 , мы получаем

    A = 2x

    B = 3Y

    Записать формулу для (формула (для (формула (для (формула (Formul a — b 3 ) в случае 2 выше.

    a 3  — b 3   =  (a — b)(a 2  + ab + b 2 )

    Подставьте 2x вместо a и 3y вместо b.

    (2x) 3  — (3y) 3   =  (2x — 3y)[(2x) 2  + (2x)(3y) + (3y) 2 ] 0 3-09 8

    0 9 1 0000 0 27y

    3 = (2x — 3y) (4x 2 + 6xy + 9y 2 )

    Вопрос 3:

    Фактор:

    125p 3 — 64Q 3

    010

    Решение:

    Запишите ( 125p 3  — 64q 3 ) в виде (a 9 0 0 9 0 9 0 9 0 0 9 0 9 0 0 0 9 0 ).

    125p 3 — 64Q 3 = (5p) 3 — (4q) 3

    (5p) 3 — (4q) 3 находится в форме (A 3 — б 3 ).

    Сравнение (a — b 3 )   и (5p) 3 — (4q) 3 , получаем

    a = 5p

    b = 4q

    Напишите формулу для (a — b 3 ), приведенную выше для случая 2.

    a 3  — b 3   =  (a — b)(a 2  + ab + b 2 )

    Замените 5p на a и 4q на b.

    (5p) 3  — (4q) 3   =  (5p — 4q)[(5p) 2  + (5p)(4q) + (4q) 2 ] 3 1

    3 1

    64кв 3 = (5p — 4q) (25p 2 + 20pq + 16q 2 )

    Вопрос 4:

    Найдите значение (M 3 — N 3 ), если m — M — n = 3 и mn = 28.

    Решение:

    Запишите (m — n 3 ) через (m — n) и mn, используя формулу, приведенную в случае 1 выше.

    m 3  — n 3   =  (m — n) 3  + 3mn(m — n)

    Подставьте 3 вместо (m — n) и 28 вместо xy.

    x 3 — Y 3 = (3) 3 + 3 (28) (3)

    x 3 — Y 3 = 27 + 252

    x 3 — Y . 3   =  279

    Помимо материалов, указанных в этом разделе, если вам нужны какие-либо другие материалы, воспользуйтесь нашим пользовательским поиском Google здесь.

    Пожалуйста, отправьте ваш отзыв на [email protected]

    Мы всегда ценим ваши отзывы.

    ©Все права защищены. онлайнmath5all.com

    a минус b Формула целого куба

    A МИНУС B ФОРМУЛА КУБА


    В этом разделе мы увидим формулу/разложение для

    (a — b) 3

    То есть, (a — b) 3  = (a — b)(a — b)(a — b)

    Умножить (a — b) и (a — b).

    (a — b) 3  = (a 2  — ab — ab + b 2 )(a — b)

    Упростить.

    (а — б) 3  = (а 2  — 2аб + б 2 ) (A — B)

    (A — B) 3 = A 3 — A 2 B — 2A 2 B + 2AB 2 + AB 2 — B 3

    Соедините подобные термины.

    (A- B) 3 = A 3 — 3A 2 B + 3AB 2 — B 3

    или

    (A- B) 3 = 3

    (A- B) 3 = 3

    b 3  — 3ab(a — b)  

    Решенные задачи


    Задача 1 :

    Развернуть :

    (x — 1) 3

    Решение :

    (x — 1) 3 в виде (a — b) 3 90 Сравнение 3 и (x — 1) 3 , получаем

    a = x

    b = 1

    Напишите формулу/разложение для (a — b) 3 .

    (a — b) 3  = a 3  — 3a 2 b + 3ab 2  — b 3

    3 90 вместо b.

    (x — 1) 3  = x 3  — 3(x 2 )(1) + 3(x)(1 2 ) — 1 3

    ) 3 = x 3 — 3x 2 + 3 (x) (1) — 1

    (x — 1) 3 = x 3 — 3x 2 + 3x — 1

    Таким образом, расширение (x — 1) 3 IS

    x 3 — 3x 2 + 3x — 1

    Проблема 2:

    Разверните:

    (2x — 3)

    .0009 3

    Решение:

    (2x — 3) 3 в форме (a — b) 3

    Сравнение 3 (a — 0 0 9)

    3 3

    , получаем

    a = 2x

    b = 3

    Напишите формулу / разложение для (a — b) 3 .

    (a — b) 3  = a 3  — 3a 2 b + 3ab 2  — b 3

    9002×2 вместо a и 9002×2.

    (2x — 3) 3  = (2x) 3  — 3(2x) 2 (3) + 3(2x)(3 2 ) — 3 3

    1 0 0 0 9 0 3 0 0 0 9 3 — 3 (4x 2 ) (3) + 3 (2x) (9) — 27

    (2x — 3) 3 = 8x 3 — 36x 2 + 54x — 27

    SO, расширение (2x — 3) 3

    8x 3 — 36x 2 + 54x — 27

    Проблема 3:

    Расширение:

    . 0003

    (x — 2y) 3

    Решение:

    (x — 2y) 3 находится в форме (A — B) 3

    Сравнение (A — B) 3 и (x — 2y) 3 , мы получаем

    a = x

    b = 2y

    Напишите формулу / разложение для (a — b) 3 .

    (a — b) 3  = a 3  — 3a 2 b + 3ab 2  — b 3

    вместо b.

    (х — 2у) 3  = х 3  — 3(х 2 )(2у) + 3(х)(2у) 2  — (2у) 3 х

    2 ) 3 = x 3 — 6x 2 Y + 3 (x) (4y 2 ) — 8y 3

    (x — 2y) 3 = x 3 — 6x 2 y + 12xy 2 + 8y 3

    Таким образом, расширение (x — 2y) 3

    x 3 — 6x 2 Y + 12xy 2  + 8y 3

    Задача 4 :

    Если a — b = 3 и a 3  — b 3  = 1197, то найдите значение ab.

    Решение:

    Чтобы найти значение ab, мы можем использовать формулу или расширение для (a — b) 3 .

    Напишите формулу/разложение для (a — b) 3 .

    (а — б) 3  = а 3  — 3а 2 б + 3аб 2  — б 3

    или

    (a — b) 3  = a 3  — b 3  — 3ab(a – b)

    Подставьте 13 вместо (a – b) и 1197 90 0 3 ).

    (3) 3  = 1197 — 3(ab)(13)

    Упрощение.

    27 = 1197 — 39ab

    Вычесть 1197 с каждой стороны.

    -1170 = -39ab

    Разделите каждую сторону на (-39).

    30 = ab

    Итак, значение ab равно 30.

    Задача 5 :

    Найдите значение:

    (98) 3

    Решение:

    Мы можем использовать алгебраическую формулу для (a — b) и легко найти значение (98) 10 0 0009.

    Напишите (98) 3 в виде (а — б) 3 .

    (98) 3  = (100 — 2) 3

    Напишите расширение для (a — b) 3 .

    (а — б) 3  = а 3  — б 3  — 3аб(а-б)

    Замените 100 на a и 2 на b.

    (100- 2) 3 = 100 3 — 2 3 — 3 (100) (2) (100- 2)

    (100- 2) 3 = 1000000- 8- 3 (100) (2) (98)

    (100- 2) 3 = 1000000- 8- 58800

    (98) 3 = 941192

    Итак, значение (107) 3

    941 192

    Пожалуйста, отправьте свой отзыв на [email protected]

    Мы всегда ценим ваши отзывы.

    ©Все права защищены. onlinemath5all.com

    Видео-урок: сумма и разность двух кубов

    Стенограмма видео

    В этом видео мы научимся разложите сумму и разность двух кубов. Мы начнем с показа формулы, которые мы можем использовать, чтобы разложить на множители или разложить на множители сумму двух кубов и разность из двух кубов.

    Многочлен в виде 𝑎 в кубе плюс 𝑏 в кубе называется суммой двух кубов. Любой полином этого вида может быть разложено так, что 𝑎 в кубе плюс 𝑏 в кубе равно 𝑎 плюс 𝑏, умноженное на 𝑎 в квадрате минус 𝑎𝑏 плюс 𝑏 в квадрате. Мы можем доказать, что это так раскрывая скобки или распределяя скобки справа сторона. Начнем с раздачи 𝑎. Умножаем 𝑎 в квадрат, минус 𝑎𝑏 и 𝑏 в квадрате на 𝑎. Это дает нам 𝑎 в кубе минус 𝑎 𝑏 в квадрате плюс 𝑎𝑏 в квадрате.

    Затем мы раздаем 𝑏. Это дает нам 𝑎 в квадрате 𝑏 минус 𝑎𝑏 в квадрате плюс 𝑏 в кубе. Мы замечаем, что можем отменить 𝑎 квадрат 𝑏 как отрицательный 𝑎 в квадрате 𝑏 плюс 𝑎 в квадрате 𝑏 равен нулю. Мы также можем отменить 𝑎𝑏 в квадрате. Это оставляет нам 𝑎 в кубе плюс 𝑏 в кубе, что равно левой части. Это доказывает, что 𝑎 в кубе плюс 𝑏 куб равен 𝑎 плюс 𝑏, умноженному на 𝑎 в квадрате минус 𝑎𝑏 плюс 𝑏 в квадрате.

    Теперь рассмотрим многочлен от форма 𝑎 в кубе минус 𝑏 в кубе. Это называется разницей два куба. 𝑎 в кубе минус 𝑏 в кубе можно разложить в следующей форме: 𝑎 минус 𝑏 умножить на 𝑎 в квадрате плюс 𝑎𝑏 плюс 𝑏 в квадрате. Еще раз, мы можем доказать это, Распределение скобок. Умножение 𝑎 на 𝑎 в квадрате плюс 𝑎𝑏 плюс 𝑏 в квадрате дает нам 𝑎 в кубе плюс 𝑎 в квадрате 𝑏 плюс 𝑎𝑏 в квадрате. Умножая три члена в вторая скобка минус 𝑏 дает нам минус 𝑎 в квадрате 𝑏 минус 𝑎𝑏 в квадрате минус 𝑏 в кубе.

    И снова 𝑎 в квадрате 𝑏 и Члены в квадрате 𝑎𝑏 отменяются, оставляя нам 𝑎 в кубе минус 𝑏 в кубе. 𝑎 в кубе минус 𝑏 в кубе равно 𝑎 минус 𝑏 умножить на 𝑎 в квадрате плюс 𝑎𝑏 плюс 𝑏 в квадрате. Во всех следующих вопросах это видео, нам нужно будет использовать одну из этих двух формул. В каждом случае мы будем переписывать соответствующую формулу, чтобы к концу этого урока мы, надеюсь, выучил их обоих.

    Учитывая, что 𝑥 в кубе минус 512 равно равно 𝑥 минус восемь умножить на 𝑥 в квадрате плюс 𝑘 плюс 64, найти выражение для 𝑘.

    Любое выражение в виде 𝑎 в кубе минус 𝑏 в кубе известен как разность двух кубов. Мы знаем, что это может быть учтено в виде 𝑎 минус 𝑏, умноженное на 𝑎 в квадрате плюс 𝑎𝑏 плюс 𝑏 в квадрате. В этом вопросе наша ценность 𝑎 в кубе равно 𝑥 в кубе, а наше значение 𝑏 в кубе равно 512. Если 𝑎 в кубе равно 𝑥 в кубе, мы знаем, что 𝑎 равно 𝑥, так как мы можем вычислить корень из обеих частей уравнения. Если 𝑏 в кубе равно 512, то 𝑏 равно восьми. Мы знаем это как восемь в кубе. равно 512, что означает, что кубический корень из 512 равен восьми.

    При разложении 𝑎 в кубе минус 𝑏 в кубе первый набор скобок содержал 𝑎 минус 𝑏. Это означает, что в нашем примере мы будет 𝑥 минус восемь. Первый член во втором множестве скобки будут 𝑥 в квадрате. Второй член равен 𝑎, умноженному на 𝑏, то есть восемь 𝑥. Последний член равен 𝑏 в квадрате, что в нашем случае восемь в квадрате, что равно 64. Нас просят найти выражение для 𝑘. Это равно восьми 𝑥. Это значение 𝑎, умноженное на 𝑏.

    В нашем следующем вопросе нам понадобится полностью разложить сумму двух кубов на множители.

    Выражение 𝑥 в кубе плюс 27 имеет два фактора. Один множитель равен 𝑥 плюс три. Что еще за фактор?

    Напомним, что любое выражение записанное в виде 𝑎 в кубе плюс 𝑏 в кубе, известно как сумма двух кубов. Это можно разделить на два набора скобок, 𝑎 плюс 𝑏 умножить на 𝑎 в квадрате минус 𝑎𝑏 плюс 𝑏 в квадрате. В этом вопросе 𝑎 в кубе равно 𝑥 в кубе и 𝑏 в кубе равно 27. Мы можем вычислить значения 𝑎 и 𝑏 путем кубического корня обеих частей этих уравнений. Это дает нам значения 𝑎 и 𝑏 из 𝑥 и трех соответственно. Теперь мы можем использовать эту информацию для множитель 𝑥 в кубе плюс 27 в два набора скобок.

    Первая скобка 𝑎 плюс 𝑏. В нашем вопросе это 𝑥 плюс три. Нам уже сказали в Вопрос, что это было одним из факторов. Наш второй набор скобок равно 𝑎 в квадрате минус 𝑎𝑏 плюс 𝑏 в квадрате. 𝑎 в квадрате равно 𝑥 в квадрате. 𝑎 умножить на 𝑏 равно три 𝑥, поэтому наш второй член равен трем отрицательным 𝑥. 𝑏 в квадрате равно девяти, так как три умножить на три будет девять. Выражение 𝑥 в кубе плюс 27 может разложить в виде 𝑥 плюс три умножить на 𝑥 в квадрате минус три 𝑥 плюс девять. Это означает, что правильный ответ на вопрос 𝑥 в квадрате минус три 𝑥 плюс девять. Это еще один фактор 𝑥 в кубе плюс 27.

    Мы могли бы проверить этот ответ, распределяя наши скобки. Мы могли бы умножить 𝑥 в квадрате минус три 𝑥 плюс девять на 𝑥, а затем умножить 𝑥 в квадрате минус три 𝑥 плюс девять на три. Когда мы это делаем, все термины отменит, за исключением 𝑥 в кубе плюс 27.

    Наш следующий вопрос более сложная задача, так как нам нужно сначала вынуть старший общий множитель.

    Полностью разложить на множители 1000𝑥 в кубе минус 125.

    На первый взгляд кажется, что это выражение записывается в виде 𝑎 в кубе минус 𝑏 в кубе, что является разность двух кубов. Мы знаем, что любое выражение этого тип может быть разложен, как показано на два набора скобок, 𝑎 минус 𝑏 умноженный на 𝑎 в квадрате плюс 𝑎𝑏 плюс 𝑏 в квадрате. Однако, когда мы рассматриваем числа 1000 и 125, мы замечаем, что у них есть наибольший общий делитель, который больше чем один. На самом деле 1000 и 125 оба делится на 125. Это означает, что мы можем начать это вопрос путем вычитания 125. 1000 разделить на 125 равно восемь. Это означает, что 125 умножить на восемь 𝑥 в кубе равны 1000 𝑥 в кубе. Так как 125 разделить на 125 равно один, 1000 𝑥 в кубе минус 125 равно 125, умноженному на восемь 𝑥 в кубе минус один.

    Выражение восемь 𝑥 в кубе минус один все еще находится в форме 𝑎 в кубе минус 𝑏 в кубе, что означает, что это может быть учитывал дальше. 𝑎 в кубе равно восьми 𝑥 в кубе, а 𝑏 в кубе равно единице. Затем мы можем кубировать корень с обеих сторон эти уравнения для расчета значений 𝑎 и 𝑏. Кубический корень из восьми равен два. Следовательно, 𝑎 равно двум 𝑥. Кубический корень из единицы равен единице, поэтому 𝑏 равен единице. Теперь мы можем факторизовать восемь 𝑥 в кубе. минус один в наши две скобки.

    𝑎 минус 𝑏 равно двум 𝑥 минус один. 𝑎 в квадрате четыре 𝑥 в квадрате два 𝑥 умножить на два 𝑥 равно четырем 𝑥 в квадрате. Умножая наши значения 𝑎 и 𝑏 дает нам два 𝑥. Наконец, 𝑏 в квадрате равно один. Восемь 𝑥 в кубе минус один, следовательно, равно двум 𝑥 минус один, умноженному на четыре 𝑥 в квадрате плюс два 𝑥 плюс один. Таким образом, мы можем заключить, что полностью факторизованная форма 1000 𝑥 в кубе минус 125 равна 125, умноженному на два 𝑥 минус один умножить на четыре 𝑥 в квадрате плюс два 𝑥 плюс один.

    В нашем следующем вопросе нам нужно вычислить сумму двух кубов, расставив скобки.

    Заполните следующее: Пробел равно 𝑦 плюс 15𝑥, умноженное на 𝑦 в квадрате минус 15𝑦𝑥 плюс 225𝑥 в квадрате.

    Наша первая мысль в этом вопросе можно попытаться распределить скобки, чтобы умножить 𝑦 в квадрате минус 15𝑦𝑥 плюс 225𝑥 в квадрате сначала на 𝑦, а затем на 15𝑥. Однако мы можем заметить, что наш выражение записывается в виде 𝑎 плюс 𝑏, умноженное на 𝑎 в квадрате минус 𝑎𝑏 плюс 𝑏 в квадрате. Это факторизованная форма выражение 𝑎 в кубе плюс 𝑏 в кубе. Это известно как сумма двух кубики. Наша ценность 𝑎 равна 𝑦, и наша значение 𝑏 равно 15𝑥.

    Мы можем вычислить значение 𝑎 в кубе и 𝑏 в кубе путем возведения в куб обеих частей каждого из этих уравнений. Наше первое уравнение дает нам 𝑎 в кубе равно 𝑦 в кубе. Кубирование обеих сторон нашего второго уравнение дает нам 𝑏 в кубе равно 3375 𝑥 в кубе. Это потому что 15 умножить на 15 умножить на 15 будет 3375. Таким образом, отсутствующий член равен 𝑦 в кубе плюс 3375 𝑥 в кубе, так как это равно 𝑦 плюс 15 𝑥, умноженное на 𝑦 в квадрате минус 15𝑦𝑥 плюс 225𝑥 в квадрате.

    Мы бы получили тот же ответ, если бы мы распределили два набора скобок. Все условия были бы отменены за исключением 𝑦, умноженного на 𝑦 в квадрате, что равно 𝑦 в кубе, и 15𝑥 умножить на 225𝑥 в квадрате, что равно 3375𝑥 в кубе.

    В нашем последнем вопросе у нас есть более сложное начальное выражение.

    Разложить на множители полностью 𝑥 минус шесть 𝑦 все в кубе минус 216𝑦 в кубе.

    Хотя может и не сразу Очевидно, это выражение записывается в виде 𝑎 в кубе минус 𝑏 в кубе. это разница двух кубики. Мы знаем, что факторизация 𝑎 в кубе минус 𝑏 в кубе равно 𝑎 минус 𝑏, умноженное на 𝑎 в квадрате плюс 𝑎𝑏 плюс 𝑏 в квадрате. Первый член в нашем выражении равен 𝑥 минус шесть 𝑦 все в кубе. Это означает, что 𝑎 в кубе равно к этому. Поскольку кубическое укоренение является противоположностью кубируя, мы можем извлекать корень из обеих частей этого уравнения, что дает нам значение 𝑎, равное до 𝑥 минус шесть 𝑦. Наш второй член равен 216𝑦 в кубе; поэтому 𝑏 в кубе равно этому. Еще раз, мы можем кубический корень оба сторон этого уравнения, что дает нам 𝑏 равно шести 𝑦, поскольку кубический корень из 216 равен шесть.

    Теперь мы можем заменить наши значения 𝑎 и 𝑏 в правую часть формулы. Начнем с 𝑎 минус 𝑏. Это равно 𝑥 минус шесть 𝑦 минус шесть 𝑦. Это упрощается до 𝑥 минус 12𝑦. 𝑎 в квадрате будет равно 𝑥 минус шесть 𝑦 все в квадрате. Это равно 𝑥 минус шесть 𝑦 умножить на 𝑥 минус шесть 𝑦. Распределяем наши скобки здесь дает нам 𝑥 в квадрате минус шесть 𝑦𝑥 минус шесть 𝑦𝑥 плюс 36𝑦 в квадрате. Это можно упростить до 𝑥 в квадрате минус 12𝑦𝑥 плюс 36𝑦 в квадрате. 𝑎𝑏 равно 𝑥 минус шесть 𝑦 умножить на шесть 𝑦. Распределение скобок здесь дает нам шесть 𝑦𝑥 минус 36𝑦 в квадрате.

    Наконец, 𝑏 в квадрате равно шести 𝑦 все в квадрате. Это равно 36𝑦 в квадрате. Заменив в замене на эти три члена дают нам 𝑥 в квадрате минус 12𝑦𝑥 плюс 36𝑦 в квадрате плюс шесть 𝑦𝑥 минус 36𝑦 в квадрате плюс 36𝑦 в квадрате. Это можно упростить до 𝑥 в квадрате минус шесть 𝑦𝑥 плюс 36𝑦 в квадрате. Шесть 𝑦𝑥 равно шести 𝑥𝑦. Таким образом, полностью факторизованный форма равна 𝑥 минус 12𝑦 умножить на 𝑥 в квадрате минус шесть 𝑥𝑦 плюс 36𝑦 в квадрате.

    Теперь мы суммируем ключ очки из этого видео. Мы можем факторизовать или факторизовать сумму двух кубов и разности двух кубов по следующим формулам. 𝑎 в кубе плюс 𝑏 в кубе равно 𝑎 плюс 𝑏 умножить на 𝑎 в квадрате минус 𝑎𝑏 плюс 𝑏 в квадрате. 𝑎 в кубе минус 𝑏 в кубе на другая рука равна 𝑎 минус 𝑏 умножить на 𝑎 в квадрате плюс 𝑎𝑏 плюс 𝑏 в квадрате. Перед использованием любого из двух формулы, важно, чтобы мы вынесли за скобки наибольший общий множитель двух термины в первую очередь.

    a куб минус b куб формула a³ — b³ Доказательства Примеры и приложения

    Что такое куб минус b куб a³ – b³?

    Формула a³ – b³ известна как формула вычитания кубов (двух чисел).

    Это просто читается как куб минус b куб. Без фактического вычисления кубов формула куб минус b куб используется для нахождения разницы между двумя кубами.

    Он также используется для факторизации биномов кубов, решения математических уравнений и многого другого.

    Формула a³ – b³ также известна как одно из важнейших алгебраических тождеств.

    Так что стоит запомнить, чтобы не застревать в ответе всякий раз, когда он требуется.

    a³ – b³ = (a² + ab + b²) (a – b)

    Выраженное словами, вычитание кубов двух чисел есть произведение разности между двумя числами на «несовершенный квадрат суммы».

    Куб минус формула куба b или формула разности кубов:

    a³ – b³ = (a² + ab + b²) (a – b)

    Алгебра включает в себя как буквы, так и цифры. Буквы или алфавиты используются для обозначения неизвестных величин в алгебраической формуле.

    Числа фиксированы, т.е. их значение известно. Теперь комбинация букв, цифр, факториалов, матриц и т. д. используется для формирования формулы или уравнения.

    Это в основном методология алгебры.

    Таким образом, можно сказать, что алгебраическая формула – это правило или уравнение, записанное с использованием алгебраических и математических выражений, состоящих из чисел, переменных (неизвестных значений) и математических функций (сложение, вычитание, деление, умножение и т. д.).

    Они в основном используются для нахождения элементов многочленов. Это уравнение, в котором алгебраические выражения с обеих сторон равны друг другу.

    Алгебраическая формула — это короткая и быстрая формула для решения сложных алгебраических вычислений.

    Есть несколько очень важных алгебраических уравнений и формул, которые студенты должны выучить перед экзаменами.

    Эти формулы являются основой элементарной или базовой алгебры.

    Здесь мы приведем список некоторых важных алгебраических формул или тождеств.

    Полный список поможет учащимся обращаться к нему в любое время или быстро просмотреть перед экзаменами.

    Советы: Одного заучивания недостаточно. Вы также должны понимать концепцию, лежащую в основе идентичности, и научиться правильно применять эти идентичности к проблеме.

    Стандартные алгебраические тождества

    Ниже перечислены четыре стандартных алгебраических тождества:

    Тождество-1: Квадрат суммы двух слагаемых

    (a + b) (a + b) или (a + b)²=b² + 2ab + a²

    Тождество-2 : Квадрат разности двух слагаемых

    (a – b) (a – b) или (a – b)² =b²–2ab + a²

    Тождество-3: Разность двух квадратов

    (a – b) ( a + b)=a² – b²

    Идентичность-4:

    (x + b) (x + a) = ab + (a + b)x +x²

    Алгебраические тождества, полученные на основе биномиальной теоремы

    Тождество-I: (a + b)² = a² + 2ab + b² + b²

    Идентичность-III: (a+b) (a–b)=a²– b²

    Идентичность- IV: (a–b)³= a³ –b³ – 3ab (a – b)

    Identity-V: (a + b + c) (a² + b² + c² – ab – bc – ca) = a³ + b³ + c³ – 3abc

    Идентичность- VI: (x + a) (x + b) = x² + (a + b)x + ab

    Идентичность- VII: (a + b + c)² = a² + b²+ c² + 2ab+2bc+ 2ac

    Тождество- VIII: (a + b)³=a³ + b³ + 3ab (a + b)

    Факторизация алгебраических тождеств

    • a² – квадрат минус b квадрат) = (a + b) (a – b)
    • a³ – b³ (Читать как куб минус b куб) = (a – b) (a² + ab + b²)
    • a³ + b³ (Читать как куб плюс b куб) = (a + b) (a² – ab +b²)
    • a⁴ – b⁴ (читается как возведение в степень 4 минус b в степени 4) = (a² – b²) (a² + b²)

    Трехчленные алгебраические тождества

    • (b + c) (a + в) (a + b) = (ac + bc + ab) (a + b + c) – abc
    • c³+b³ + a³ – 3abc = (a² + b² + c² – ab– bc – ca) (a + b + c)
    • c² + b² + a² = (a + b + c)² – 2 ( ac + bc + ba)
    • (a – b) (a – c) = bc – (b + c) a + a²

    Советы по запоминанию алгебраической формулы:-

    • Используйте количество каналов, чтобы выучить формулу.
    • Заранее ознакомьтесь с формулой. Это отличная идея, чтобы прочитать предстоящие темы в вашем учебнике, прежде чем идти в школу.
    • Не отвлекайтесь.
    • Ежедневное выполнение задач поможет вам запомнить формулы.

    Часто ученики путаются между выражениями и идентичностями. Ниже изложены некоторые различия между ними.

    • Алгебраическое выражение может быть определено как математическое выражение, состоящее из переменных/чисел, включая арифметическую операцию между ними, в то время как алгебраические тождества представляют собой связь эквивалентности, такую ​​как X=Y. Это означает, что X и Y включают некоторые переменные. Кроме того, X и Y дают одно и то же значение, несмотря на значения, которые покрываются для переменной.
    • Алгебраическое выражение в общем случае представляет собой выражение, состоящее из констант и переменных, причем переменная может принимать любое значение. Следовательно, значение выражения может измениться, если значения переменных изменены/изменены/переключены. Наоборот, алгебраическое тождество верно для всех значений переменных.
    • Например, 6q + 3 — это алгебраическое выражение, где 6q и 3 — термы, а «q» обозначает переменную предоставленного выражения. Здесь переменные и термины разделены арифметическим знаком/символом +. Давайте поймем разницу между алгебраическим выражением и тождествами с помощью другого примера: 8x + 5 = 20, где 8x + 5 — левая часть (LHS) тождества, а 20 — правая часть (RHS). личности. Знак равенства между левой и правой сторонами символизирует, что значение левой стороны идентично правой части идентификатора.

    Давайте докажем, верна ли эта формула a³-b³ = (a-b) (a²+ab+b²)? Для этого нам нужно доказать, что левая сторона (левая сторона) = правая сторона (правая сторона). Давайте начнем наше доказательство с шагов, упомянутых ниже: —

    LHS (левая сторона) = a³ – b³ (читается как куб минус b куб)

    При решении правой стороны мы получаем,

    = (b² + ba + a²) (a – b)

    При умножении a и b по отдельности на (a²+ ab + b²) мы получаем,

    = a (b² + ba+ a²) – b(b² + ba + a²)

    = b²a + a²b – b³ + a³ – ba² – ab²

    = a²b – a²b + ab²- ab² + a³ -b³ [путем объединения одинаковых членов и исключения возможных членов, таких как ba² и b²a в правой части Термин]

    = A³ — 0 — B³

    = A³ — B³

    Следовательно, доказано, что LHS = RHS

    Давайте также докажем это, положив

    A = 5 и B = 3THEN, 69 A = 5 и B = 3THEN, 69 A = 5 и B = 3THEN, 9 A = 5 и B = 3THEN, 9

    A = 5 и B = 3THEN, 9 A = 5 и B = 3.

    LHS = a³ – b³

    LHS = 5³-3³

    LHS = 125 – 27

    левая сторона = 98

    правая сторона = (a – b)(a² + b²+ ab)

    правая сторона = (5-3)(5²+3²+ 5×3)

    правая сторона = (2)(25+9 +15)

    RHS = 98

    ∴ LHS = RHS, Отсюда доказано.

    Как мы знаем,

    (a−b)³ =a³ – b³ – 3a²b + 3ab²

    ⇒a³ – b³ =(a – b)³ + 3a²b – 3ab²

    ⇒a³ – b³ = 3ba (a – b) +(a – b)³

    Теперь, взяв (a – b) общее у нас есть,

    ⇒a³-b³ =(a−b)[(a−b)²+3ab]

    Поскольку, мы знаем, что

     (a−b)² = a²+b²−2ab

    Используя это свойство в приведенном выше уравнении, получаем –

    ⇒ a³ – b³ = (3ab – 2ab + b² + a²) (a – b)

    Упрощая приведенное выше уравнение, получаем –

    ⇒ a³ – b³ = (ba + b² + a²) (a – b)

    Следовательно, это искомая формула a³-b³.

    Примечание- Формулу для (a−b)³ можно легко получить, написав (a−b)³ = (a−b)(a−b)². Затем, используя тождество, то есть (a−b)² = a²+b²−2ab

    Итак, (a−b)³ = (a−b)(a²+b²−2ab)

    Упрощение этой правой части поможет нам получить значение (a−b)³. Всякий раз, когда мы сталкиваемся с трудностями в запоминании этой формулы, описанные выше шаги помогают вывести ее немедленно.

    Ответив на несколько хороших вопросов, вы сможете лучше понять формулу a³ – b³= (b² + ba + a²) (a – b).

    Пример 1: Найдите значение 100³- 8³, используя эту формулу.

    Решение: В этом вопросе нам нужно найти: 100³- 8³. Здесь мы будем считать, что a = 100 и b = 8. Поэтому мы заменим эти a и b на 100 и 8 соответственно в формуле a³ – b³= (b² + ba + a²) (a – b)

    100³-8³ = (100-8)(100²+100*8+8²)

    = (92) (10000 + 800 +64)

    = (92)(10864)

    =9999989

    3

    3 Ответ: 999488

    Пример 2: Факторизируйте следующее выражение 8p³- 27, используя формулу a³-b³= (b² + ba + a²) (a – b).

    Решение: Здесь мы должны разложить на множители: 8p³- 27. Мы будем использовать эту формулу, чтобы разложить это выражение на множители. Предположим, что a = 2p и b = 3. Таким образом, мы можем переписать данное выражение в виде a³-b³.

    8p³- 27 = (2p)³- 3³

    Так как, a³-b³= (b² + ba + a²) (a – b)

    (2p)³-3³ =(2p-3)((2p) ²+(2p)(3)+3²)

    = (2p-3) (4p²+6p+9)

    Ответ: (2p-3) (4p²+6p+9)

    Пример 3: Если a-b=5,ab=20, найти a³-b³ ?

    Решение : Так как (a – b) = 5

    Кубирование обеих сторон, мы получаем,

    (a-b)³ = 5³

    a³-b³-3ab(a-b) = 000 9025 из (a – b) и ab, получаем

    aждать-B³-3*20 (5) = 125

    aждать-B³-300 = 125

    aждать-B³ = 125 + 300

    = 425

    Ответ: A³-B³ = 425 .

    Пример 4: Факторизируйте выражение 27x³– 125, используя формулу a³-b³.

    Решение : Разложить на множители: 27x³ – 125.

    Мы можем записать данное выражение как формула a³– b³.

    a³– b³ = (b² + ba + a²) (a – b)

    (3x)³-5³ =(3x-5)((3x)²+(3x)(5)+5²)

    = (3x-5) (9x²+15x+25)

    Ответ: (3x-5) (9x²+15x+25)

    • Составление расписания занятий
    • Игра в ловлю мяча, в которую играют 5- 6-летние дети
    • Ребенок, развивающий пространственный интеллект
    • Компьютерное программирование
    • Астрологические расчеты
    • Покупки
    • Нахождение налогового обязательства
    • Preparing the food or halving/doubling the recipe
    • Technological developments
    • Budgeting
    • Doing Interiors and Landscape designing
    • Professional Sports
    • Real Estate Project Planning

    Frequently Asked Questions:

    • How To Use the a³– b³ Формула?

    Следующие шаги выполняются при использовании формулы a³–b³. Во-первых, следует отметить структуру Nos, независимо от того, имеют ли они ^3 в качестве силы или нет. Запишите формулу a³– b³. 92» означает «а» в произведении с самим собой дважды. Это очень простое обозначение. А «2а» означает, что у вас есть переменная «а», просуммированная сама с собой 2 раза. В первом случае это умножение указанного числа на себя, а в другом случае — сложение числа с самим собой.

    • Как упростить числа с помощью формулы a³ – b³?

    Давайте разберемся в использовании формулы a3 – b3 с помощью следующего примера: Найдите значение 10³ – 3³, используя формулу a³ – b³. Предположим, что a = 10 и b = 3. Подставим их в формулу a³– b³.

    а³ – b³= (b² + ba + a²) (a – b)

    10³-3³ = (10-3)(10²+ (10)(3)+3²)

    = (7) (100+ 30+9)

    = (7)(139)

    = 973

    • Каковы основные правила алгебры?

    Распределительные, ассоциативные и коммутативные законы являются основными законами алгебры. Они помогают в определении неизвестных величин, объясняют взаимосвязь между числовыми операциями и способствуют упрощению уравнений или их решению.

    • Есть ли разница между (a³-b³) и (a-b)³. Если я пишу оба как одно и то же. Есть ли ошибка? Что там за ошибка?

    Да, между (a³-b³) и (a-b)³ огромная разница.

    (a – b)³ [Читать как минус b целый куб] = a³ – 3ba(a – b) – b³

    Тогда a³ – b³ = 3ba (a – b) + (a – b)³

    = (a – b)[(a – b)² + 3ab]

    = (a–b)(a² – 2ab + b² + 3ab)

    = (b² + ba + a²) (a – b)

    Читать Еще

    Учебник по математике

    Специальная факторизация: суммы и разности кубов и полные квадраты

    Дифф. of SquaresPerfect-Square Tri’sRecognizing Patterns

    Purplemath

    Две другие специальные формулы факторинга, которые вам нужно запомнить, очень похожи друг на друга; это формулы для факторизации сумм и разностей кубов. Вот две формулы:

    Факторизация суммы кубов:

    a 3 + B 3 = ( A + B ) ( A 2 AB + B 2 )

    Факторинг. B 3 = ( A B ) ( A 2 + AB + B 2 )

    с этими формулами. А пока просто запомните их.

    Содержание продолжается ниже

    MathHelp.com

    Чтобы облегчить запоминание, сначала обратите внимание, что термины в каждой из двух формул факторизации абсолютно одинаковы. Затем обратите внимание, что каждая формула имеет только один знак «минус». Различие между двумя формулами заключается в расположении этого одного знака «минус»:

    Для разности кубов знак «минус» идет в линейном множителе, a b ; на сумму кубов, знак «минус» стоит в квадратном множителе, a 2 ab + b 2 .

    Некоторые люди используют мнемонику « SOAP «, чтобы отслеживать знаки; буквы обозначают линейный множитель, имеющий «тот же» знак, что и знак в середине исходного выражения, затем квадратичный множитель, начинающийся со знаком «противоположный» тому, что был в исходном выражении, и, наконец, второй знак внутри исходного выражения. квадратичный множитель «всегда положителен».

    A 3 ± B 3 = ( A [ То же самое ] B ) ( A 2 [ OAPPOSION ). Положительный ] b 2 )

    Какой бы метод ни лучше помог вам сохранить эти формулы в чистоте, используйте его, потому что вы не должны предполагать, что вам дадут эти формулы на тесте. Вы должны ожидать, что должны знать их.

    Примечание. Квадратичная часть каждой формулы куба не учитывает , поэтому не тратьте время на ее факторизацию. Да, A 2 — 2 AB + B 2 и A 2 + 2 AB + B 2 Фактор, но это из -за 2 -го их в середине. условия. Квадратичные члены этих формул суммы и разности кубов не имеют , что «2», и, таким образом, не может множить.


    Когда вам дали пару кубиков для разложения, тщательно применяйте соответствующее правило. Под «осторожно» я подразумеваю «использование круглых скобок, чтобы отслеживать все, особенно отрицательные знаки». Вот несколько типичных проблем:

    • Коэффициент
      x 3 − 8

    Это эквивалентно x 3 − 2 3 . Со знаком «минус» посередине это разность кубов. Чтобы сделать факторинг, я буду подключать x и 2 в формулу разности кубов. Это я получаю:

    x 3 — 8 = x 3 — 2 3

    = ( x — 2) ( x 2 + 2 x ). + 2 2 )

    = ( x − 2)( x 2 + 2 x + 4)


    • Коэффициент 27
      x 3 + 1

    Первый член содержит куб 3 и куб x . Но как насчет второго срока?

    Прежде чем паниковать из-за отсутствия видимого куба, я вспомнил, что 1 можно рассматривать как возведенную в любую степень, которую я захочу, поскольку 1 в любой степени по-прежнему равно 1. В этом случае степень, которую я хотел бы, равна 3 , так как это даст мне сумму кубов. Это означает, что выражение, которое они мне дали, может быть выражено как:

    (3 x ) 3 + 1 3

    Итак, чтобы разложить на множители, я подставлю 3 x и 1 в формулу суммы кубов. Это дает мне:

    27 x 3 + 1 = (3 x ) 3 + 1 3

    = (3 x + 1) (3 x ) 2 x + 1) (3 x ) 2. — (3 x ) (1) + 1 2 )

    = (3 x + 1) (9 x 2 — 3 x + 1)


    • Фактор
      x 3 y 6 − 64

    Во-первых, я отмечаю, что они дали мне бином (многочлен с двумя членами) и что степень x в первом члене равна 3, поэтому, даже если бы я не работал в «суммах и различия кубов» моего учебника, я бы обратил внимание, что, возможно, мне следует думать в терминах этих формул.

    Глядя на другую переменную, я замечаю, что степень числа 6 является кубом степени числа 2, поэтому другая переменная в первом члене также может быть выражена в терминах куба; а именно, как куб квадрата г .

    Второе слагаемое равно 64, которое, как я помню, является кубом числа 4. (Если бы я не помнил или не был уверен, я бы схватил свой калькулятор и попытался вычислить что-то в кубе, пока не получил правильное значение, иначе я бы взял кубический корень из 64.)

    Теперь я знаю, что с «минусом» посередине это разница в два куба; а именно, это:

    ( xy 2 ) 3 − 4 3

    Подставляя в соответствующую формулу, я получаю:

    x 3 Y 6 — 64 = ( XY 2 ) 3 — 4 3

    = ( XY 2

    = ( XY 2

    = ( XY 2

    = ( XY 2

    = ( XY 2

    = ( XY 2

    = ( XY 2

    = ( xy 2

    . 2 ) 2 + ( xy 2 )(4) + 4 2 )

    = ( xy 2 − 4)( x 2 y 4 + 4 xy 2 + 16)


    Эм… Я знаю, что 16 равно не куб чего угодно; на самом деле он равен 2 4 . Как дела?

    Дело в том, что они ожидают, что я использую то, что я узнал о простом факторинге, чтобы сначала преобразовать это в разность кубов. Да, 16 = 2 4 , но 8 = 2 3 , куб. Я могу получить 8 из 16, разделив на 2. Что произойдет, если я разделю 250 на 2? Я получаю 125, что является кубом 5. То, что они мне дали, можно переформулировать как:

    2(2 3 x 3 -5 3 )

    I может применить формулу разницы в кубиках к тому, что находится внутри скобков:

    2 3 x 3 -5 3 = (2 x ) -5 3 = (2 x ) -5 3 = (2 x )0-5 3 = (2 x ) -5 3 = (2 x ) 3 — (5) 3

    = (2 x — 5)((2 x ) 2 + (2 x )(5) + (5) 0 0 2

    9 2 2 = (2 x − 5)(4 x 2 + 10 x + 25)

    Собрав все вместе, я получаю окончательную факторизованную форму:

    2(2 x − 5)(4 x 2 + 10 x + 25)


    • Коэффициент −
      x 3 − 125.

    Моей первой реакцией могло бы быть прямое применение формулы разности кубов, поскольку 125 = 5 3 . А что насчет того знака «минус» впереди?

    Поскольку ни одна из формул факторинга, которые они мне дали, не включает «минус» впереди, может быть, я могу вынести «минус» из множителя…?

    х 3 — 125 = -1 х 3 — 125

    = -1( х 3 0 + 125 га Теперь то, что находится внутри скобок, представляет собой сумму кубов, которую я могу разложить на множители. У меня есть сумма куба x и куба 5, поэтому:

    x 3 + 5 3 = ( x + 5)(( x

    ) 2 900
    — ( х )(5) + (5) 2 )

    = ( х + 5)( х 2 — 5 х + 25)

    Собрав все вместе, я получаю:

    -1( х + 9015 х 5)( х 9 0)( х 9 0)( х 9 0 5 x + 25)


    Вы можете использовать виджет Mathway ниже, чтобы попрактиковаться в факторизации суммы кубов.

    Добавить комментарий

    Ваш адрес email не будет опубликован. Обязательные поля помечены *